SlideShare a Scribd company logo
.
1
VISION IAS
www.visionias.in
TEST BOOKLET
CSAT APTITUDE TEST– (4025) – 2023
Time Allowed: Two Hours Maximum Marks: 200
INSTRUCTIONS
1. IMMEDIATELY AFTER THE COMMENCEMENT OF THE EXAMINATION, YOU SHOULD CHECK THAT THIS BOOKLET
DOES NOT HAVE ANY UNPRINTED OR TORN OR MISSING PAGES OR ITEMS ETC. IF SO, GET IT REPLACED BY A
COMPLETE TEST BOOKLET.
2. ENCODE CLEARLY THE TEST BOOKLET SERIES A, B, C OR D AS THE CASE MAY BE IN THE APPROPRIATE PLACE IN
THE ANSWER SHEET.
3. You have to enter your Roll Number on the Test Booklet in
the Box provided alongside. DO NOT write anything else on
the Test Booklet.
4. This Test Booklet contains 80 items (Questions). Each item is printed in English. Each item comprises four
responses (answers). You will select the response which you want to mark on the Answer Sheet. In case you feel
that there is more than one correct response, mark the response which you consider most appropriate. In any
case, choose ONLY ONE response for each item.
5. You have to mark all your responses ONLY on the separate Answer Sheet provided. See direction in the answers
sheet.
6. All items carry equal marks. Attempt all items. Your total marks will depend only on the number of correct
responses marked by you in the answer sheet. For every incorrect response one-third of the allotted Marks will
be deducted.
7. Before you proceed to mark in the Answer sheet the response to various items in the Test booklet, you have to
fill in some particulars in the answer sheets as per the instruction sent to you with your Admission Certificate.
8. After you have completed filling in all responses on the answer sheet and the examination has concluded, you
should hand over to Invigilator only the answer sheet. You are permitted to take away with you the Test
Booklet.
9. Sheets for rough work are appended in the Test Booklet at the end.
DO NOT OPEN THIS BOOKLET UNTIL YOU ARE ASKED TO DO SO
C
.
2 www.visionias.in ©Vision IAS
1. A tailor has to cut multiple rolls of fabric with a
pair of scissors. Each roll of fabric has to be cut
into 10 equal pieces. If he makes 45 cuts with
his scissors per minute, how many rolls would
be cut in 24 minutes?
(a) 30 rolls
(b) 58 rolls
(c) 120 rolls
(d) 150 rolls
2. A girl started counting the fingers of her left
hand in the following manner: She started by
calling the thumb 1, the index finger 2, middle
finger 3, ring finger 4, little finger 5, and then
reversed direction of counting calling the ring
finger 6, middle finger 7 and so on. If she
counted until 1994, she must have ended
counting on which finger?
(a) Ring finger
(b) Middle finger
(c) Thumb
(d) Index finger
3. Choose a figure which would most closely
resemble the unfolded form of Figure (Z).
Select the answer using the correct code.
(a) 1
(b) 2
(c) 3
(d) 4
4. Pinky was born on 29th Feb, 2016 which
happened to be a Monday. On what day would
she celebrate her birthday next?
(a) Monday
(b) Tuesday
(c) Friday
(d) Saturday
5. Find out which of the figures (1), (2), (3) and
(4) can be formed from the pieces given in
figure (X).
(a) 1
(b) 2
(c) 3
(d) 4
6. What was the day of the week on 16th June,
1999?
(a) Saturday
(b) Monday
(c) Wednesday
(d) Thursday
7. A man has Rs. 480 in the denominations of one
rupee notes, five rupee notes and ten rupee
notes. The number of notes of each
denomination is equal. What is the total number
of notes that he has?
(a) 45
(b) 75
(c) 90
(d) 120
.
3 www.visionias.in ©Vision IAS
Directions for the following 6 (six) items:
Read the following five passages and answer the items
that follow each passage. Your answers to these items
should be based on the passages only.
Passage – 1
New Zealand Cricket striking a deal to remunerate its
women cricketers the same as their male counterparts is
a major landmark in the fight to close the gender pay
gap in sports. The United States women’s national
footballers won six yearlong battles with their
federation to secure equal compensation. The
agreements are expected to be game changers,
encouraging more girls to take up the sports. As New
Zealand captain Sophie Devine said, “It is great to be
recognized in the same agreement, alongside the men. It
is a massive step forward and will be a huge drawcard
for young women and girls.” But victories in equal pay
struggles have not come easy. In cricket, any move to
narrow the monetary gap between men and women,
especially in India, is dithered over by citing lower
market ratings for the ladies’ game. But it would be
prudent to focus on the factors that are holding women
back — unequal opportunities, curtailed playing time
and lack of investment. Historically, men taking to
sports and following sports have been organic
exercises, largely because of social conditioning.
8. Which one of the following statements best
reflects the critical message conveyed by the
passage given above?
(a) Achieving gender pay parity is an
important step towards achieving gender
equality, but challenges of social
conditioning remain.
(b) The need of the hour is to improve the
social condition of women.
(c) Reducing the pay gap is about being fair
and respectful, and recognizing the effort
and excellence women bring to the sport.
(d) It is time the journey towards pay parity is
hastened.
9. With reference to the above passage, the
following assumptions have been made:
1. Society has nudged women to internalize
that sporting participation is not for them,
vis-à-vis men.
2. For women, it is trivial to be recognized at
the same level as men.
Which of the above assumptions are valid?
(a) 1 only
(b) 2 only
(c) 1 and 2
(d) None
Passage – 2
The capital city of the world’s largest democracy is
driven by power, energy, and opportunities, dipped in
nostalgia, rich in history and heritage and soaked in a
myriad of compelling identities, memories, and
emotions. The composite culture of thousands of years
thrives even after the city was destroyed and rebuilt
seven times. Each time it has risen like a phoenix from
the ashes stamping the city with its genetic code
distinctly seen in its architecture, monuments,
museums, food, art, poetry, politics, culture, and
language. The built environment of the city is a product
of its changing socio-economic, cultural, and political
forces. Those who live and work here lend to the city’s
unique character, but its pollution levels, the game of
one-upmanship, chaos and scare can be frustrating and
infuriating. Yet, the city is counted among the most
desirable ones to live in. Something is alluring and
magical about it.
10. According to the passage, which among the
following constructs a narrative of the city
mentioned?
1. Symbolic imagery that addresses the
dormant social, political and cultural
climate.
2. Long eventful history of the city that saw
the rise and fall of several empires.
Which of the statements given above is/are
correct?
(a) 1 only
(b) 2 only
(c) 1 and 2
(d) None
.
4 www.visionias.in ©Vision IAS
Passage—3
In terms of things we keep, things that encapsulate
particular moments of our past, it all depends on your
relationship to memory, to how much you feel the need
to live in the past, or how you use the past to fuel your
present and future. I know people with vivid memories
who have no need for the triggers or proof provided by
objects. I also know people who have huge drop-outs in
their memory but who won’t let go of a single dust-
encrusted trinket. As for myself, all I know is that I
have a newly deployed drawer for fresh hoarding and
lots of magical elbowing and adjusting going on in
other drawers as newly arrived things jostle with ones
that have been occupying the space for years.
11. According to the passage, which of the
following is correct about things we keep as
souvenir?
1. They are necessary to remind people
suffering from memory loss, about past
events.
2. They keep lying idle in some drawers for
years and serve no useful purpose.
3. They are most likely kept by those people
who like to relive their pasts.
Select the correct answer using the code given
below.
(a) 2 and 3 only
(b) 3 only
(c) 1 and 2 only
(d) 1, 2 and 3
Passage – 4
The Governor of the Reserve Bank of India (RBI) has
been reported as saying that the repo rate has been
raised. Raising the interest rate in an attempt to control
inflation, implicitly assumes that it reflects economy
wide excess demand. To persist with monetary policy
to curb inflation under these circumstances is to miss
the point that, being a macroeconomic instrument, it
cannot affect any particular price. raising interest rates
is not going to solve the problem of inflation. It is not
going to create more food. What you do is that you
have supply-side interventions. Killing the economy
through raising interest rates is not going to solve the
inflation in any time frame. It is a common mistake to
observe sharply rising prices of certain goods and
conclude therefrom that it is this that is driving
inflation. This conclusion can be way off the mark when
the concerned goods account for only a small part of the
consumption basket that the overall consumer price
index is based on.
12. Which among the following is the most logical
and rational inference that can be made from
the passage given above?
(a) Economists tend to pay adequate attention
to supply side interventions.
(b) Rather than correcting inflation by raising
the interest rates, focus should be more on
necessary supply-side interventions.
(c) India’s economic agencies appear to have
not fully understood what is driving
inflation.
(d) Monetary policy to curb inflation can never
be effective.
Passage – 5
Landslides are a common occurrence during the rainy
season in hilly and mountainous regions. Heavy or
continuous rain usually causes mud and rocks to come
sliding down the mountain/hill, and in the process,
destroy everything along their path. Houses, vehicles,
people, trees, and animals often get buried in a
landslide. When someone has a landslide victory in an
election, he wins the contest by an overwhelming
margin; he defeats his rivals soundly. What is it that a
landslide does? It buries everything along its path.
Someone who wins an election by a landslide destroys
everything in his path; in this case, it suggests that the
candidate does not just beat his opponents, he buries
them!
13. Which one of the following statements best
sums up the passage given above?
(a) Sliding down the mountain/hill reflects
winning over the opponent.
(b) While it is true that a ‘landslide’ is usually
associated with misfortune, a ‘landslide
victory’ is seen as something evil.
(c) While a ‘landslide’ is usually associated
with something bad, during elections a big
victory is called a landslide victory.
(d) There is an elementary correlation between
landslides in a mountainous region and big
victories during the election.
.
5 www.visionias.in ©Vision IAS
14. Six friends – A, B, C, D, E and F are sitting in
two rows, three in each row. Each boy is sitting
exactly opposite to another boy. A is to the
North of D, and to the East of E. B is to the
South-East of E and to South-West of A. F is
not sitting to the West of B. Who is sitting in
between C and D?
(a) A
(b) B
(c) E
(d) F
15. A clock is set right at 10 a.m. on Monday. It
loses 15 minutes in 24 hrs. What will be the
true time when the clock indicates 4 a.m. on the
following Saturday?
(a) 5:12 am
(b) 5:32 am
(c) 6:32 am
(d) 5:48 am
16. The minute hand of a faulty clock overtakes the
hour hand at intervals of 63 minutes of correct
time. How many minutes in a day does the
clock lose or gain?
(a) 58(4/71) min
(b) 54(6/81) min
(c) 55(7/70) min
(d) 56(8/77) min
17. In 2016, Mohan celebrated his birthday on
Friday. Which will be the first year after 2016
when Mohan will celebrate his birthday on a
Wednesday? (He was not born in January or
February)
(a) 2021
(b) 2023
(c) 2020
(d) 2025
Directions for following 2 (two) items:
Consider the information given below and answer the
items that follow.
In a circus there are seven animals – Lion, Tiger, Deer,
Rabbit, Cow, Giraffe and Crocodile. Lion, Tiger and
Crocodile are carnivores; Deer, Rabbit, Cow and
Giraffe are herbivores. For a play, a team of four
animals has to be selected. Out of four, not more than
two can be carnivores, and at least two must be
herbivores.
Further there are following restrictions to be kept in
mind:
1. If Crocodile is included, then both Giraffe and Rabbit
are excluded.
2. Lion and Tiger cannot be included together.
3. If Cow is included, then Deer cannot be included.
18. Which one of the following combinations is not
possible?
(a) Tiger, Giraffe, Deer and Rabbit
(b) Lion, Cow, Giraffe and Rabbit
(c) Crocodile, Cow, Rabbit and Tiger
(d) Lion, Rabbit, Cow and Giraffe
19. If Tiger, Giraffe and Deer are selected, then
which among the following can be the fourth
member of the team?
(a) Cow
(b) Lion
(c) Crocodile
(d) Rabbit
.
6 www.visionias.in ©Vision IAS
Directions for the following 3 (three) items:
Consider the information given below and answer the
items that follow.
Recently, Ghosh Babu spent his winter vacation on
Kyakya Island. During the vacation, he visited the local
casino where he came across a new card game. Two
players, using a normal deck of 52 playing cards, play
this game. One player is called the Dealer and the other
is called the Player.
First, the Player picks a card at random from the deck.
This is called the base card. The amount in rupees equal
to the face value of the base card is called the base
amount. The face values of Ace, King, Queen and Jack
are ten. For other cards, the face value is the number on
the card.
Once, the Player picks a card from the deck, the Dealer
pays him the base amount. Then the Dealer picks a card
from the deck and this card is called the top card.
* If the top card is of the same suit as the base card, the
Player pays twice the base amount to the Dealer.
* If the top card is of the same colour as the base card
(but not the same suit), then the Player pays the base
amount to the Dealer.
* If the top card happens to be of a different colour than
the base card, the Dealer pays the base amount to the
Player.
Ghosh Babu played the game 4 times.
* The first time he picked eight of clubs and the Dealer
picked queen of clubs.
* Second time, he picked ten of hearts and the dealer
picked two of spades.
* Next time, Ghosh Babu picked six of diamonds and
the dealer picked ace of hearts.
* Lastly, he picked eight of spades and the dealer
picked jack of spades.
Answer the following questions based on these four
rounds.
20. What could have been the maximum gain made
by Ghosh Babu, if he stopped playing at the
right time (instead of playing all the four
rounds)?
(a) 12
(b) 20
(c) 16
(d) 4
21. The initial money that Ghosh Babu had was Rs.
X (before the beginning of the game rounds).
At no point did he have to borrow any money.
What is the minimum possible value of X?
(a) 16
(b) 8
(c) 100
(d) 24
22. If the final amount of money that Ghosh Babu
had with him was Rs.100, what was the initial
amount he had with him?
(a) Rs. 120
(b) Rs. 8
(c) Rs. 4
(d) Rs. 96
Directions for the following 3 (three) items:
Read the following three passages and answer the items
that follow each passage. Your answers to these items
should be based on the passages only.
Passage—1
We are now in the middle of a long process of
transition in the nature of the image which man has of
himself and his environment. Primitive men, and to a
large extent also men of the early civilizations,
imagined themselves to be living on a virtually
illimitable plane. That is, there was always someplace
else to go when things got too difficult, either by reason
of the deterioration of the natural environment or a
deterioration of the social structure in places where
people happened to live. Gradually, however, man has
been accustoming himself to the notion of the spherical
earth and a closed sphere of human activity where there
is nowhere to go if things get difficult.
.
T
h
i
s
d
o
c
u
m
e
n
t
i
s
p
e
r
s
o
n
a
l
i
s
e
d
f
o
r
A
t
i
s
h
S
h
r
e
y
a
n
s
h
(
a
t
i
s
h
v
i
s
i
o
n
2
0
0
2
@
g
m
a
i
l
.
c
o
m
)
7 www.visionias.in ©Vision IAS
23. With reference to the above passage, following
assumptions have been made:
1. Only rise in population alters man’s image
of himself and his environment.
2. Primitive men lacked the capability to
accustom themselves to changed external
situations – material or non-material.
Which of the above assumptions is/are valid?
(a) 1 only
(b) 2 only
(c) Both 1 and 2
(d) Neither 1 nor 2
Passage – 2
The discussion on the demerits of ‘freebies’ distributed
to the public as a result of election promises is not new
in India. However, there is often confusion on what
constitutes ‘freebies,’ with several services that the
Government provides to meet its constitutional
obligations towards citizens also being clubbed in this
category. The basic argument is that these are a waste
of resources and place a burden on already stressed
fiscal resources. In such discussions, ‘freebies’ not only
include the free distribution of what may be considered
‘club goods’ such as televisions and gold chains but
also welfare schemes such as free or subsidized rations
under the Public Distribution System (PDS), cooked
meals under the mid-day meal scheme, supplementary
nutrition through anganwadis, and work provided
through the Mahatma Gandhi National Rural
Employment Guarantee Act (MGNREGA).
MGNREGA for instance has been another scheme
which has been a lifeline for many during the pandemic
and earlier. At a time when there are few employment
opportunities, working under MGNREGA can
guarantee some assured wages; implemented in the true
spirit of the legislation.
24. Concerning the above passage, the following
assumptions have been made:
1. Making welfare delivery an electoral issue
is a strategy adopted by politicians.
2. Freebies have both a positive and negative
impact on the socio-economic growth of a
nation.
3. Subsidy burden of the state also contributes
to human development and protection of
the basic rights of the people to nutrition,
work and essentially the right to life with
dignity.
Which of the above assumptions is/are valid?
(a) 1 only
(b) 2 and 3 only
(c) 3 only
(d) 1, 2, and 3
Passage – 3
Violations of the right to education may occur through
the direct action of States parties (the act of
commission) or through their failure to take steps
required by law (the act of omission). Whilst the vast
majority of countries have ratified international treaties
that recognize the full right to education, it is still
denied to millions around the world due to a lack of
resources, capacity, and political will. There are still
countries that have not integrated the right to education
into their national constitution or provided the
legislative and administrative frameworks to ensure that
the right to education is realized in practice. Most of the
children and adults who do not fully enjoy the right to
education belong to the most deprived and marginalised
groups of society which are often left behind in national
policies.
.
8 www.visionias.in ©Vision IAS
25. Based on the above passage, the following
assumptions have been made:
1. Integration of the right to education in
national laws will end the violations of the
right to education.
2. Violations of the right to education can be
done by state parties.
Which of the above assumptions is/are valid?
(a) 1 only
(b) 2 only
(c) 1 and 2 only
(d) None
26. Tuesday fell on which of the following dates of
April, 2002?
(a) 3rd, 10th, 17th, 24th
(b) 1st, 8th, 15th, 22nd, 29th
(c) 4th, 11th, 18th, 25th
(d) 2nd, 9th, 16th, 23rd, 30th
27. Consider the following figure.
Find the minimum number of straight lines
required to make the given figure.
(a) 28
(b) 26
(c) 23
(d) 24
28. Sheetal went to a ‘Sale’ to purchase some pants
and skirts. When her friends asked her how
many of each she had bought, she answered,
“The number of skirts is two less than twice the
number of pants purchased. Also, the number
of skirts is four less than four times the number
of pants purchased”. What is the number of
pant and skirts purchased by Sheetal?
(a) 2, 2
(b) 1, 0
(c) 1, 1
(d) 4, 2
29. Consider the following pattern of numbers:
16 18 22 26
8 9 10 13
20 16 10 11
28 25 22 ?
What is the missing number in the above
pattern?
(a) 19
(b) 24
(c) 26
(d) 28
30. The time in a clock is 20 minute past 2. Find
the angle between the hands of the clock.
(a) 60 degrees
(b) 120 degrees
(c) 45 degrees
(d) 50 degrees
31. How often between 11 O'clock and 12 O'clock
are the hands of the clock together?
(a) 5
(b) 2
(c) 3
(d) 0
.
9 www.visionias.in ©Vision IAS
32. 27th February, 2003 was a Thursday. What
must have been the day on 27th February,
1603?
(a) Monday
(b) Thursday
(c) Sunday
(d) Tuesday
33. A clock loses 1% of the week-time during the
first week and then gains 2% of the week-time
during the next one week. If the clock was set
right at 12 noon on a Sunday, what will be the
time that the clock will show exactly 14 days
from the time it was set right?
(a) 1: 36
(b) 1: 40
(c) 1: 41
(d) 1: 19
34. Select the option in which the given figure is
embedded. (Rotation or Flipping is not
allowed).
(a)
(b)
(c)
(d)
Directions for the following 4 (four) items:
Consider the information given below and answer the
items that follow.
Cinemax is a very popular multiplex in the city of
Mumbai. It has five screens – Screen 1, Screen
2, Screen 3, Screen 4 and Screen 5.
Five different movies – Predestination (PN), Looper
(LR), Interstellar (IR), Inception (IN) and Equalizer
(ER) – were screened over five consecutive days on
these screens. Each movie was shown on a different
screen every day, i.e. every movie was shown on each
screen exactly once. Similarly, each screen showed a
different movie every day, i.e. every screen showed a
particular movie exactly once.
Further, the following information is known:
i. In all the screens, except screen 2, IR was screened
just before IN.
ii. LR was the third movie to be screened on screen 4,
and it was screened two days after PN.
iii. PN was screened on two other screens prior to
screen 3, and was screened on 1 immediately after it
was screened on 3.
iv. IN was released on screen 3 on day 2.
v. On day 1, PN and ER were released on two
consecutive screen numbers.
35. Which movie was screened on the first day on
screen 2?
(a) ER
(b) IN
(c) PN
(d) Cannot be determined
36. On which screen was IR screened on the fourth
day?
(a) Screen 4
(b) Screen 1
(c) Screen 2
(d) Cannot be determined
.
10 www.visionias.in ©Vision IAS
37. Which of the following is the correct order of
screens (from first day to fifth day) on which
Equalizer was screened?
(a) 4, 1, 2, 3, 5
(b) 3, 5, 1, 2, 4
(c) 5, 4, 2, 3, 1
(d) Cannot be determined
38. On which screen was IN screened immediately
on the next day after it was screened on screen
1?
(a) 4
(b) 5
(c) 2
(d) Cannot be determined
Directions for the following 3 (three) items:
Read the following two passages and answer the items
that follow each passage. Your answers to these items
should be based on the passages only.
Passage – 1
Climate change and its impacts across the globe will
threaten the bottom line of businesses in a variety of
ways. The frequency and intensity of extreme weather
in countries, can damage factories, supply chain
operations and other infrastructure, and disrupt
transport. Drought will make water more expensive,
which will likely affect the cost of raw materials and
production. Climate volatility may force companies to
deal with uncertainty in the price of resources for
production, energy transport and insurance. And some
products could become obsolete or lose their markets,
such as equipment related to coal mining or skiing in an
area that no longer has snow. Whether in the U.S. or
abroad, new regulations such as carbon pricing and
subsidies that favour a competitor may affect a
business’s bottom line. A company’s reputation could
also suffer if it’s seen as doing something that hurts the
environment. And investors and stakeholders are
increasingly worried about the potential for “stranded
assets”—those that become prematurely obsolete or fall
out of favour and must be recorded as a loss, such as
fossil fuels that many believe should stay in the ground
or real estate in a newly designated flood plain.
39. Which among the following are the possible
disadvantages of climate change?
1. Increased prices of goods.
2. Forcing companies to think about the
environment.
3. Redundancy of products.
4. Introduction of carbon pricing.
Select the correct answer from the code given
below:
(a) 1 and 3 only
(b) 3 and 4 only
(c) 2 and 4 only
(d) 1 and 2 only
40. Which one of the following statements best
reflects the crux of the passage?
(a) Climate change will drive the change in the
pattern of future production of resources.
(b) Companies which no longer focus on
climate governance will face a boycott of
their goods.
(c) Nations across the globe need to tackle
climate change by making it their number
one priority.
(d) Only focusing on sustainable development
can help us cope with the menace of
climate change.
Passage—2
The root of sufferings is neither the feeling of pain nor
of sadness nor even of meaninglessness. Rather, the real
root of suffering is this never-ending and pointless
pursuit of ephemeral feelings, which causes us to be in
a constant state of tension, restlessness and
dissatisfaction. Due to this pursuit, the mind is never
satisfied. Even when experiencing pleasure, it is not
content, because it fears this feeling might soon
disappear, and craves that this feeling should stay and
intensify. People are liberated from sufferings not when
they experience this or that fleeting pleasure, but rather
when they understand the impermanent nature of all
their feelings, and stop craving for them.
.
11 www.visionias.in ©Vision IAS
41. What is the most logical and rational
inference that can be made from the above
passage?
(a) As feelings are never-ending and
ephemeral, sufferings can never end and
human mind cannot ever be satisfied.
(b) Feelings of pain, sadness and
meaninglessness, not in isolation, but in
combinations of two or three are at the root
of sufferings.
(c) Gautam Buddha’s observations on
sufferings in this world are universal and
timeless.
(d) None of the above
42. What is the angle between the minute hand and
the hour hand when the time is 15:40 hours?
(a) 150
(b) 160
(c) 140
(d) 130
43. The first two problem figures given below
follow a certain pattern. The same pattern
should be followed by the next two problem
figures. Choose the correct answer figure that
can replace the question mark.
Problem figures
Answer figures
(a) 1
(b) 2
(c) 3
(d) 4
44. Out of the four figures marked (a) (b) (c) and
(d), three are similar in a certain manner.
However, one figure is not like the other three.
Choose the figure which is different from the
rest.
(a) a
(b) b
(c) c
(d) d
Directions for the following 4 (four) items:
Consider the information given below and answer the
items that follow.
Kumar has a peach orchard in Ooty. On a particular
day, his associate Suresh plucked 103 distinct peaches
(each peach of a distinct size). He placed these 103
peaches in four distinct boxes – A, B, C and D at 6 AM.
Later at multiple times during the day (8 AM, 10 AM,
12 noon and 2 PM), Kumar tested some of the peaches
in some or all of the four boxes. He performed one of
the following three operations – I, II and III on the
peaches he tested:
Operation I: If Kumar liked any peach in a box, he
plucked a new peach of the same size and placed it in
one of the other three boxes.
Operation II: If Kumar did not like the box in which a
particular peach was placed, he removed the peach
from that box and placed it in another box.
Operation III: If Kumar did not like a particular peach,
he removed the peach from the box and threw it away.
The following table provides information about the
number of peaches in each of the boxes – A, B, C and
D at different times during the entire day. Kumar was
very lazy and hence performed minimum possible
number of operations. An operation is said to have been
performed, when any of I, II or III is performed.
Boxes 6 AM 8 AM 10 AM 12 Noon 2 PM
A 23 25 29 26 28
B 27 31 31 32 29
C 15 27 40 39 32
D 38 30 25 24 45
.
12 www.visionias.in ©Vision IAS
45. Find the total number of operations performed
by Kumar between 6:00 AM and 2:00 PM.
(a) 62
(b) 67
(c) 63
(d) 65
46. How many times did Kumar perform operation
III between 6:00 AM and 2:00 PM?
(a) 6
(b) 4
(c) 5
(d) 3
47. At least how many times was operation II
performed to remove a peach from box D and
place it in box C between 6:00 AM and 2:00
PM?
(a) 1
(b) 2
(c) 3
(d) More than 3
48. How many times did Kumar perform operation
II between 6:00 AM to 2:00 PM?
(a) 24
(b) 25
(c) 23
(d) 22
Directions for the following 4 (four) items:
Read the following two passages and answer the items
that follow each passage. Your answers to these items
should be based on the passages only.
Passage – 1
With the rise of packaged food, food with high salt and
sugar content, obesity-related malnutrition is on the
rise. India is now facing a double whammy of
malnutrition, with the younger population being
affected by undernutrition and the older population
being affected by malnutrition related to obesity. We
currently have no significant programmes that target
malnutrition affected by obesity. As a country, we
could take the community nutrition approach where
nutrition is improved at the community level. One of
the most successful examples of this approach in the
country is the addition of iodine to salt to prevent goitre
and intellectual disabilities. This approach includes
educating people about dietary changes and helping
them achieve optimum levels of nutrition. An important
part of this is educating children at schools and people
in shared spaces such as workplaces. The main aim of
this approach is preventive.
49. Based on the above passage, the following
assumptions have been made:
1. It is not possible to overcome malnutrition
by obesity as community nutrition is
preventive in nature.
2. Absence of government programmes for
fighting malnutrition is the primary cause
of its existence.
3. People should be encouraged to eat fresh
and nutrition rich food instead of packaged
or high-salt foods.
Which of the above assumptions is/are valid?
(a) 1 and 3 only
(b) 3 only
(c) 1, 2 and 3
(d) 1 and 2 only
.
13 www.visionias.in ©Vision IAS
50. Which one of the following statements best
reflects the message of the author of the
passage?
(a) With the success of fighting goitre, the
community nutrition approach is the best
approach for fighting malnutrition by
obesity.
(b) Considering the ill consequences, India
should focus on handling malnutrition by
obesity using the community nutrition
approach.
(c) Government needs to devise more effective
programmes for fighting malnutrition by
obesity.
(d) Companies manufacturing packaged food,
food with high salt and sugar content
should be adequately regulated.
Passage – 2
Invasive species threaten biodiversity globally, and
invasive mammalian predators are particularly
damaging, having contributed to considerable species
decline and extinction. Invasive predators are
implicated in 87 birds, 45 mammals, and 10 reptile
species extinctions - 58% of these groups’
contemporary extinctions worldwide. These figures are
likely underestimated because 23 critically endangered
species that were assessed are classed as “possibly
extinct.” Invasive mammalian predators endanger a
further 596 species at risk of extinction, with cats,
rodents, dogs, and pigs threatening the most species
overall. Species most at risk from predators have high
evolutionary distinctiveness and inhabit insular
environments. Invasive mammalian predators are
therefore important drivers of the irreversible loss of
phylogenetic diversity worldwide. That most impacted
species are insular indicates that management of
invasive predators on islands should be a global
conservation priority.
51. Which of the following is/are the most rational
and logical Inference/Inferences that can be
made from the passage?
1. Invasive species along with being
dangerous for the environment also, have
some potential benefits for it.
2. Island species are the most vulnerable to
extinction due to invasive predators.
Select the correct answer from the code given
below:
(a) 1 only
(b) 2 only
(c) Both 1 and 2
(d) Neither 1 nor 2
52. Which one of the following statements best
reflects the crux of the passage?
(a) To ensure the survival of the human race,
the conservation of biodiversity is the most
important task.
(b) Countries across the world should fund
biodiversity conservation efforts on Islands.
(c) Invasive species can grow anywhere and
are always endangering the endemic
species.
(d) Invasive predators have the potential to
change the species diversity of an
ecosystem.
53. In a caravan, in addition to 50 hens, there are
45 goats, 8 camels and some human keepers. If
the total number of feet are 224 more than the
number of heads, the number of keepers must
be
(a) 5
(b) 8
(c) 10
(d) 15
.
14 www.visionias.in ©Vision IAS
54. Aruna cuts a cake into two halves and then
further cuts one of the halves into smaller
pieces of equal size. Each of the smaller pieces
is twenty grams in weight. If she has seven
pieces of the cake in all with her, what must
have been the total weight of the original cake?
(a) 100 grams
(b) 150 grams
(c) 200 grams
(d) 240 grams
55. At what time between 1 and 2 o’ clock will the
hands of a watch make an angle of 180°?
(a) 35(5/11) min. past 1
(b) 40 min. past 1
(c) 50(4/11) min. past 1
(d) 38(2/11) min. past 1
56. Which of the answer figures can replace the
question mark in the problem figure?
(a) D
(b) C
(c) A
(d) B
57. Out of the given answer figures, which is the
correct one to replace the empty box?
(a) a
(b) b
(c) c
(d) d
58. If 28th August, 1946 was a Wednesday; what
day of the week must have fallen on 31st
August, 1961?
(a) Tuesday
(b) Thursday
(c) Monday
(d) Wednesday
59. If Palmyra < Date plam; Date plam < Queen
palm and Queen palm > Caryota, then which of
the following is always true?
(a) Palmyra > Caryota
(b) Palmyra < Queen palm
(c) Date palm = Caryota
(d) Date palm > Caryota
.
15 www.visionias.in ©Vision IAS
Direction for the following 3 (three) items:
Consider the information given below and answer the
items that follow.
In a school, there are exactly six classrooms in a
straight line. In these classrooms six different classes
are held, namely for class I, II, III, IV, V and VI. In the
first period exactly six different subject teachers -
Hindi, English, Science, Math, Drawing and Sports -
went to these classes, such that only one teacher went to
each class.
The following information is also known about the
teachers who went to the various classes:
1. Drawing teacher went to the class to the immediate
left of class V and to the immediate right of class II.
2. Math teacher went to class III which is adjacent to
class II.
3. English teacher went to the class to the immediate
left of class I.
4. Hindi teacher went to class IV, which is first from
the left end and to the immediate left of the class to
which Science teacher went.
5. There are exactly four classes to the left of class l and
class IV is not adjacent to class III.
60. Math teacher went to which class?
(a) V
(b) VI
(c) III
(d) II
61. Which subject teacher went to class VI?
(a) Science
(b) Hindi
(c) Drawing
(d) Sports
62. Which subject’s teacher went to the class,
which is to the immediate right of class II?
(a) Science
(b) Hindi
(c) Drawing
(d) Math
Directions for the following 4 (four) items:
Read the following three passages and answer the items
that follow each passage. Your answers to these items
should be based on the passages only.
Passage – 1
The food system and agriculture are deeply interrelated
with land, water, and energy. Agriculture in the U.S. is
an energy-intensive sector and a major user of ground
and surface water. Direct energy use in agriculture is
primarily from fossil fuels used to operate automobiles
and machinery for preparing fields, planting, and
harvesting crops, applying chemicals, and transporting
inputs and outputs to and from the market. Electricity is
used largely for lighting, heating, cooling, and
operation of in-farm equipment. Indirect energy is
consumed off the farm for manufacturing fertilizers and
pesticides, with the production of fertilizer being
“extremely energy-intensive, requiring large amounts of
natural gas”. Currently, there are U.S. public programs
that promote renewable energy options and
recommendations in conserving fuel and energy for
farms, efficient use of fertilizers and pesticides, soil
conservation measures, and energy-efficient methods of
growing and transporting foods, but those programs
will need high levels of participation to curb both
climate impacts as well as address air and water quality
issues associated with traditional agricultural practices.
63. Based on the above passage, the following
assumptions have been made:
1. Future of agriculture in the U.S. is bleak
due to dependence on fossil fuels.
2. Only the replacement of fossil fuels with
renewable energy may not solve the
problem of air and water pollution caused
by agriculture.
Which of the above assumptions is/are valid?
(a) 1 only
(b) 2 only
(c) Both 1 and 2
(d) Neither 1 nor 2
.
16 www.visionias.in ©Vision IAS
64. Which among the following are the possible
uses of energy in agriculture?
1. Preparation of fields for growing crops.
2. Transportation of inputs to and outputs
from the field.
3. Application of fertilizers.
Select the correct answer from the code given
below:
(a) 1, 2 and 3
(b) 1 and 2 only
(c) 2 and 3 only
(d) 1 and 3 only
Passage - 2
A National Green Tribunal (NGT) bench headed by
Justice Sudhir Agarwal on September 28, 2022 ordered
mining operations at the Shaliganga Nallah in the
Budgam district of the Kashmir Valley to halt, calling
into question the environmental clearances (EC) given
for the operations. The order stated that according to the
appellant, “All the three proposals were rejected by the
(administration-appointed) Jammu and Kashmir Expert
Appraisal Committee (JKEAC) on the ground that the
area for which ECs were applied, are already
overexploited; depleted to a large extent due to heavy
illegal mining.” “Shaliganga Nallah is a feeding
channel for the Hokersar wetland and mining activity
may be detrimental to natural flow and quality of water
flowing into Hokersar; irrigation Kuhls are also present
within the mining site,” it added. Kuhls are traditional,
community-managed irrigation systems in Kashmir and
Himachal Pradesh. The JKEAC examined the mining
blocks on the latest Google Image of October 2021. It
found that minerals had been depleted to a large extent
due to heavy illegal mining. Hardly any material was
available for exploitation without endangering the
environmental setting of the Nallah.
65. Which of the following is/are the most
rational and logical Inference/ Inferences
that can be made from the passage?
1. Technology is assisting the government
with environmental protection.
2. The Himalayas' traditional and locally run
irrigation system is endangered by mining.
3. Water contamination and impairment of
water flow may be caused by mining
activities.
Select the correct answer using the codes given
below.
(a) 2 only
(b) 2 and 3 only
(c) 3 only
(d) 1, 2 and 3
Passage – 3
In just the last three centuries, global forest areas have
shrunk by 40 per cent. Every year, to meet the timber
needs from natural sources, the Earth is stripped of 100
million trees. They store 50 per cent of the world’s
terrestrial carbon and provide a buffer from extreme
weather, such as hurricanes and tsunamis. The “State of
the World’s Trees” assessment evaluated 60,000 tree
species and found that 30 per cent are at the risk of
extinction. Extinction in the plant kingdom is “twice the
number of threatened tree species globally than
threatened mammals, birds, amphibians and reptiles
combined”. Over 440 tree species are on the brink of
extinction, meaning they have fewer than 50 individuals
remaining in the wild, the report reveals. The rapidly
snowballing loss in diversity is largely triggered and
fuelled by new age emergencies and interventions, like
climate change, change in atmospheric composition and
the spread of invasive alien species. The current
warming is 1.2°C above the pre-industrial levels. This
means, from the evolutionary perspective, humans are
already moving out of the Holocene environment that
ensured the right temperature for us to evolve and take
up farming. The IPCC report cites that half of all
species are moving towards the poles or to a higher
elevation to adapt to the new planetary climate. At the
sea, due to the warming, species have travelled
poleward at the rate of 59 km per decade on average.
.
17 www.visionias.in ©Vision IAS
66. Biodiversity loss is due to which of the
following factors?
1. Spread of alien invasive species
2. Global warming and climate change
3. Deforestation
Which of the above assumptions is/are valid?
(a) 1 and 2 only
(b) 2 and 3 only
(c) 1 and 3 only
(d) 1, 2 and 3
67. A bus starts from city X. The number of
women in the bus is half of the number of men.
In city Y, 10 men leave the bus and five women
enter. Now the number of men and women
becomes equal. In the beginning, how many
passengers entered the bus in city X?
(a) 25
(b) 35
(c) 45
(d) 55
68. Consider the following equation:
5# × 3#0 ÷ #0 – 3#3 + #3 = 3#
In the above equation what does the symbol #
stands for?
(a) 4
(b) 5
(c) 8
(d) 6
69. What is the number of days in x weeks and x
days?
(a) 7x + 1
(b) 8x
(c) 15x
(d) 7x + 7
70. If yesterday were tomorrow, then today would
be Friday. What day is today?
(a) Saturday
(b) Monday
(c) Sunday
(d) Tuesday
Directions for the following 4 (four) items:
Consider the information given below and answer the
items that follow.
O, P, Q, R, S, T, U, V, W, X, Y and Z are 12 persons, 3
each from four different families – the Ahuja’s, the
Sharma’s, the Malhotra’s and the Mishra’s. Given
below are five groups, each group consisting of four
persons, such that no group consists of two persons
from the same family.
Group A O Q R S
Group B Q W Y Z
Group C R T V X
Group D O U V Z
Group E P U X Y
Further it is given that:
I. R and Y are not from the same family.
II. T and Y are not from the same family.
III. O is from the Ahuja’s.
IV. Neither T nor Y are from the Sharma’s.
V. T is not from the Ahuja’s.
71. X is from the same family as:
(a) Q
(b) W
(c) Z
(d) Cannot be determined
.
18 www.visionias.in ©Vision IAS
72. Who among the following is definitely from the
Sharma’s?
(a) Z
(b) P
(c) Both Z and P
(d) either Z or P
73. Which of the following statements is not
correct?
(a) If Q is from the Mishra’s, then S is from
the Malhotra’s.
(b) X is from the Ahuja’s and Z is from the
Sharma’s.
(c) If U is from the Malhotra’s, then S is from
the Mishra’s.
(d) None of these.
74. For how many persons, the exact families they
belong to can be uniquely determined?
(a) 10
(b) 9
(c) 8
(d) 6
75. When Rahul was born, his father was 32 years
older than his brother and his mother was 25
years older than his sister. If Rahul’s brother is
6 years elder to him and his mother is 3 years
younger than his father, then how old was
Rahul’s sister when he was born?
(a) 5 years
(b) 8 years
(c) 10 years
(d) 14 years
Directions for the following 5 (five) items:
Read the following four passages and answer the items
that follow each passage. Your answers to these items
should be based on the passages only.
Passage – 1
The societal pressure to pursue happiness ironically
seems to have adverse effects on people's psychological
well-being. Especially in countries that score high on
the World Happiness Index (WHI), there is a strong
link between the feeling of needing to be happy and the
extent to which people actually experience feelings
such as sadness, gloom, fatigue or anxiety. 'The pursuit
of happiness' prevails everywhere, on social media, in
self-help books and in advertisements. Because you
only see seemingly happy people, it is extra
conspicuous if you deviate from the happiness norm.
As a result, people who don't meet the standard can
have a more negative self-image and feel even worse.
Negative emotions often seem unacceptable. There is
still a stigma on sadness, grief, and fear.
76. On the basis of the passage given above, the
following assumptions have been made:
1. In countries that score low on the World
Happiness Index (WHI), there is a weak
link between the feeling of needing to be
happy and the extent to which negative
emotions are experienced.
2. Achieving societal norms of happiness will
lead to happiness in life.
Which of the above assumptions is/are valid?
(a) 1 only
(b) 2 only
(c) 1 and 2 both
(d) Neither 1 nor 2
.
19 www.visionias.in ©Vision IAS
Passage – 2
The principle that patients should be autonomous is a
central tenet of medical law and ethics. Autonomy
gives rise to certain interests and certain rights: the
interests of self-determination and bodily integrity, and
the moral and legal right to consent to and to refuse
medical treatment. Naturally, the autonomous cosmetic
surgery patient shares this interest in self-determination
and bodily integrity and thus, the moral and legal right
to consent to and to refuse treatment. The patient should
be able to refuse treatment, but how far should the
doctor be able to refuse to give treatment asked for?
Ideas of autonomy have not only been related to the
individual self-expression of the patient; indeed, they
have been used to validate arguments for clinical
independence and discretion. O’Neill has argued,
following Kant, that autonomy goes beyond
individualism and self-expression and involves the
exercise of duties, obligations, and respect for other
moral agents and their own rights.
77. Which of the following statements best reflects
the message of the author of the passage?
(a) Every patient has the right to get treated by
the doctor.
(b) Patients are moral agents that exercise their
rights over their body, and such autonomy
can never be violated.
(c) Medical law and ethics attempts to
reconcile the dichotomy between patient's
rights and doctor's rights and clinical
independence.
(d) In medical law and ethics, bodily integrity,
self-determination, self-expression and
individualism are cardinal.
Passage - 3
The United Arab Emirates’s (UAE)’s Advanced Visa
System, first announced by the Federal Authority for
Identity, Citizenship, Customs and Ports Security (ICP)
last month, will come into effect on Monday (October
3). Approved by UAE’s Cabinet in April, the new visa
rules — aimed at reforming the country’s immigration
and residency policies — include changes such as
longer visas for tourists, extended residency for
professionals under the Green Visa and an expanded
10-year Golden Visa scheme. Besides alleviating
burdens for foreigners and simplifying visa procedures,
the updated visa system is an attempt at “improving the
quality of life and making the experience of living,
working and investing in the UAE a pleasant and happy
one”, said Major General Sultan Yousef Al Nuaimi, the
Director General of Residency and Foreigners Affairs,
as reported by Gulf News. Tourist visas will now allow
visitors to legally enter and stay in the UAE for 60
days, an increase from the previous 30 days. A five-
year, flexible multi-entry tourist visa was also
introduced that allows them to stay in the UAE for up
to 90 days in a row. Furthermore, the job exploration
visa, which seeks to easily allow talented professionals
to find employment in the UAE, will not require a
sponsor or host. Those that fall under the first, second
or third skill level classification of the Ministry of
Human Resources and Emiratisation and new graduates
from the top 500 universities of the world will be
eligible for the job exploration visa.
.
20 www.visionias.in ©Vision IAS
78. Which of the following is/are the most rational
and logical Inference/ Inferences that can be
made from the passage?
1. The new visa system was put into place in
order to promote increased investment in
the UAE.
2. Foreigners can stay in the UAE for up to 4
months straight, thanks to the introduction
of a multi-entry tourist visa.
Select the correct answer using the codes given
below.
(a) 1 only
(b) 2 only
(c) Both 1 and 2
(d) Neither 1 nor 2
Passage – 4
Since there are multiple truths and multiple
perspectives in sociology, it is hardly surprising that
there are also multiple methods. There is no single
unique road to sociological truth. Of course, different
methods are more or less suited to tackle different types
of research questions. Moreover, every method has its
own strengths and weaknesses. It is thus futile to argue
about the superiority or inferiority of different methods.
It is more important to ask if the method chosen is the
appropriate one for answering the question that is being
asked. For example, if one is interested in finding out
whether most Indian families are still ‘joint families’,
then a census or survey is the best method. However, if
one wishes to compare the status of women in joint and
nuclear families, then interviews, case studies or
participant observation may all be appropriate methods.
There are different ways of classifying or categorising
various methods commonly used by sociologists. It is
conventional, for example, to distinguish between
quantitative and qualitative methods: the former deals
in countable or measurable variables (proportions,
averages, and the like) while the latter deals with more
abstract and hard to measure phenomena like attitudes,
emotions and so on. A related distinction is between
methods that study observable behaviour and those that
study non-observable meanings, values and other
interpretational things.
79. Which of the following is/are the most rational
and logical Inference/ Inferences that can be
made from the passage?
1. Participant observation is the best method
to use if one wants to determine whether
the majority of Indian families are still
considered "joint families."
2. There is no single, objective method in
sociology because there are many
competing truths and perspectives.
Select the correct answer using the codes given
below.
(a) 1 only
(b) 2 only
(c) Both 1 and 2
(d) Neither 1 nor 2
80. Which one of the following is the most
important implication to the above passage?
(a) It's crucial to select the one right research
approach to get accurate sociological
findings.
(b) The path to sociological truth is not one-
size-fits-all.
(c) The status of women has improved in
nuclear families.
(d) For the most portion, qualitative research
methods are used in sociological
procedures.
.
1 www.visionias.in ©Vision IAS
VISIONIAS
www.visionias.in
ANSWERS & EXPLANATION
APTITUDE TEST–Test (4025) – 2023
1.(c)
Number of cuts needed to cut a roll into 10 pieces = 9.
Therefore, Number of rolls cut = 24 × 45/9 = 120.
2.(d)
Clearly, while counting, the numbers associated with the thumb are 1, 9, 17, 25,………, i.e., numbers of
the form (8n + 1).
Since 1994 = 249 × 8 + 2, so 1993 shall correspond to the thumb and 1994 to the index finger.
3.(a)
4.(d)
29th Feb, 2016 = Monday. So, 28th Feb, 2016 = Sunday
28th Feb, 2017 = Tuesday (because 2016 is a leap year, there will be 2 odd days)
Therefore, Feb 28th
2018 (Wednesday), Feb 28th 2019 (Thursday), Feb 28th 2020 (Friday), Feb 29th 2020
(Saturday)
5.(c)
6.(c)
16th
June, 1999 = (1998 years + time period from 1.1.1999 to 16.6.1998)
1600 years » 0 odd days
300 years » 1 odd day
98 years has 74 ordinary years + 24 leap years.
Number of odd days in 98 years (24 × 2 + 74) = 122 days = 3 odd days.
Now, January + February + March + April + May + June of 1999 = (31 + 28 + 31 + 30 + 31 + 15) = 166
days = 23 weeks & 5 days = 5 odd days.
So, Total number of odd days = (0 + 1 + 3 + 5) = 2
So, the given day is Monday + 2 = Wednesday.
7.(c)
Let the number of notes of each denomination be x.
Then, x + 5x + 10x = 480
Or 16x = 480
Or x = 30
Hence, total number of notes = 3x = 90
8.(a)
Option (a) is correct. The passage mentions, “striking a deal to remunerate its women cricketers the
same as their male counterparts is a major landmark in the fight to close the gender pay gap in sports.,”
“But it would be prudent to focus on the factors that are holding women back — unequal opportunities,
.
2 www.visionias.in ©Vision IAS
curtailed playing time and lack of investment. Historically, men taking to sport and following sport have
been organic exercises, largely because of social conditioning.” It implies achieving gender pay parity is
an important step towards gender equality. But the social conditioning is still holding them back. So, this
option is correct.
Option (b) is incorrect. The statement only talks about improving the social condition of women. It does
not bring about the issue of the gender pay gap. So, this option does not best capture the essence of the
passage in its entirety. Therefore, this option is incorrect.
Option (c) is incorrect. The statement only recognizes the change brought about by a reduced gender pay
gap. It does not incorporate social conditioning facets of women. So, this statement is incorrect.
Option (d) is incorrect. This statement only talks about pay parity. The passage is more holistic in nature
when it comes to gender-gap issue.
9.(a)
Assumption (1) is correct. The passage mentions, “In cricket, any move to narrow the monetary gap
between men and women, especially in India, is dithered over by citing lower market ratings for the
ladies’ game. But it would be prudent to focus on the factors that are holding women back — unequal
opportunities, curtailed playing time and lack of investment. Historically, men taking to sport and
following sport have been organic exercises, largely because of social conditioning.” It implies, through
social conditioning, women have been strained to internalize that sporting participation is not that
significant an activity for them. So, this assumption is correct.
Assumption (2) is incorrect. The passage mentions, “It’s great to be recognized in the same agreement,
alongside the men.” It highlights the importance of placing men and women at the same pedestal. As
“trivial” signifies something of little value, this assumption is incorrect.
10.(b)
Statement (1) is incorrect. The passage mentions, “The built environment of the city is a product of its
changing socio­economic, cultural, and political forces.” Here, the author mentioned “changing”, not
“dormant” or “stagnant”, social, political, and cultural climate. So, this option is incorrect.
Statement (2) is correct. The passage mentions, “The composite culture of thousands of years thrives
even after the city was destroyed and rebuilt seven times. Each time it has risen like a phoenix from the
ashes stamping the city”. It implies a long eventful history of several rises and falls. This statement builds
the narrative of the city. So, this option is correct.
11.(b)
Souvenir: a thing that is kept as a reminder of a person, place, or event.
Statement 1 is incorrect. The passage does not limit its scope to a section of people suffering from
memory loss.
Statement 2 is incorrect as the passage nowhere says that these souvenirs serve no purpose. The passage in
the last sentence just says that they “have been OCCUPYING THE SPACE for years”. Also, the tone of
this passage is not against the practice of keeping such things. Author quotes potential advantages of such
souvenirs like their role in helping us use the past to fuel our present and future.
Statement 3 is correct. The passage in the 1st
sentence says that “it all depends on your relationship to
memory…” and “ to how much you feel the need to live in the past”. Hence, it is likely that people
desirous of reliving their pasts keep such things.
12.(b)
Option (a) is incorrect. In the passage, the author is suggesting that more attention is paid to managing
interest rates via the monetary policy, expressing the need to instead focus upon supply-side interventions
in the macroeconomic policy. So, this option is incorrect.
Option (b) is correct. The passage’s main purpose is to bring to light how the misconception that raising
the interest rates would cure inflation is leading to a lack of attention to necessary supply-side
interventions. So, this is the most logical and rational inference that can be made from the passage. So,
this option is correct.
.
3 www.visionias.in ©Vision IAS
Option (c) is incorrect. This option is partially incorrect, because the passage nowhere questions the
understanding of the economic agencies. It only questions the focus and steps being taken, suggesting a
re-direction of efforts. So, this option is incorrect.
Option (d) is incorrect. This option is not correct. Monetary policy might not be effective if it leaves out
supply-side interventions. But we cannot infer that monetary policy can never be effective in curbing
inflation. This passage is suggesting that “To persist with monetary policy to curb inflation under these
circumstances is to miss the point”. So, this option is incorrect.
13.(c)
Option (a) is incorrect. The statement is absurd. So, this option is incorrect.
Option (b) is incorrect. The passage mentions, “in the process, destroy everything along their path.
Houses, vehicles, people, trees, and animals often get buried in a landslide.,” “When someone has a
landslide victory in an election, he wins the contest by an overwhelming margin; he defeats his rivals
soundly.” It implies, landslides bring unfortunate circumstances along with them, destroy everything in
their path etc. The author compared a landslide to an election victory with an overwhelming margin,
which leaves opponents quite far back. This kind of victory is good, and it should be celebrated. So, this
option is incorrect.
Option (c) is correct. The passage mentions, “Heavy or continuous rain usually causes mud and rocks to
come sliding down the mountain/hill, and in the process, destroy everything along their path,” and “When
someone has a landslide victory in an election, he wins the contest by an overwhelming margin; he
defeats his rivals soundly.” The author compared a landslide to a big victory in the passage. While a
landslide is a misfortune, a landslide victory is a celebration. So, this option is correct.
Option (d) is incorrect. The passage does not suggest that there is any correlation between landslides in a
mountainous region and big victories during the election.
14.(b)
A is to the North of D and to the East of E. B is to the South–East of E and South–West of A.
The above data can be represented as follows:
F is not sitting to the West of B. Hence, F must be to the North of B, and C to the West of B.
Hence, the final arrangement will be:
Therefore, B is sitting in between C and D.
15.(a)
From 10 am on 1st day to 4 am on 5th day there are a total of 114 hours.
As the clock loses 15 min per day, so 23 hours 45 min of this clock are the same as 24 hours of correct
clock, i.e. 95/4 hours of this clock = 24 hours of correct clock.
So, 114 hours of this clock = (24×4/95) × 114 hours of correct clock = 115.2 hours of correct clock, which
is equal to 115 hours 12 minutes.
So, the correct time must be 5:12 am.
16.(d)
In a correct clock, the minute hand gains 55 min spaces over the hour hand in 60 min.
To be together again, the minute hand must gain 60 min over the hour hand.
60 min are gained in (60/55) × 60 = 65(5/11) min
But they are together after 63 min.
So, gain in 63 min = 65 (5/11) – 63 = 2(5/11) = (27/11) min.
So, gain in 24 hrs = [27/(11 × 63)] × 60 × 24 = 56(8/77) min.
.
4 www.visionias.in ©Vision IAS
17.(c)
Since it has been mentioned that Mohan was not born in February, so his date of birth can’t be 29th Feb.
He will celebrate his next birthday on a Wednesday in the year by which the number of odd days becomes
5.
By his birthday in 2017, there will be 1 odd day.
By his birthday in 2018, there will be 2 odd days.
By his birthday in 2019, there will be 3 odd days.
By his birthday in 2020, there will be 5 odd days, as 2020 is a leap year.
So, in 2020 he will celebrate his birthday on a Wednesday.
18.(c)
According to the question, out of four, not more than two can be carnivores and at least two must be
herbivores. It means that the possible number of carnivores and herbivores may be: (2, 2), (1, 3), (0, 4).
Further it’s given that:
Statement 1: Crocodile ↮ Giraffe and Rabbit
Statement 2: Lion ↮ Tiger
Statement 3: Cow ↮ Deer
The combination given in option (c) is not possible because Crocodile cannot be included with Rabbit.
19.(d)
Option (a) cannot be the answer because Cow cannot be included with Deer.
Option (b) cannot be the answer because Lion cannot be included with Tiger.
Option (c) cannot be the answer because Crocodile cannot be included with Giraffe.
Hence, option (d) is the required answer.
20.(a)
There are 3 scenarios:
(1) If the top card is of the same suit as the base card, the Player pays twice the base amount to the Dealer.
(2) If the top card is of the same colour as the base card (but not the same suit) then the Player pays the
base amount to the Dealer.
(3) If the top card happens to be of a different colour than the base card, the Dealer pays the base amount
to the Player.
Now Ghosh babu played four rounds of the game. Let us list down the total amount with him after each
round.
Round 1: 8 - 16 = -8 (from case 1)
Round 2: (-8) + 10 + 10 =12 (from case 3)
Round 3: (+12) + 6 - 6 =12 (from case 2)
Round 4: (+12) + 8 —16 = 4 (from case 1)
Hence, we can conclude that by round 2 and 3 he had the maximum profit of Rs 12.
21.(b)
In the previous solution we have seen that maximum loss he incurred was Rs 8 at the end of 1st round.
Hence, if he had Rs 8 he wouldn’t need to borrow money from anyone.
22.(d)
With Rs. X initially, he is left with Rs. (X+4) at the end.
Hence, X + 4 = 100
Or X = 96
23.(d)
Statement 1 is incorrect. The passage says, primitive men imagined themselves to be living on a virtually
illimitable plane, whereas presently the concepts of spherical earth and a closed sphere of human activity
are being explored. We cannot conclusively say that this change in perception has been driven ONLY by
rise in population; rather it may be because of the newly gained scientific knowledge regarding our planet.
.
5 www.visionias.in ©Vision IAS
Statement 2 is also incorrect. Author only says that primitive men imagined themselves to be living on a
virtually illimitable plane. There is no direct indication towards whether or not they had the capability to
adjust to changed circumstances. Secondly, the passage suggests that there has been a gradual change in
perception of human beings. This would not be possible had primitive men lacked adjusting capability.
24.(d)
Assumption (1) is correct. The passage mentions, “Freebies’ distributed to the public as a result of
election promises are not new in India.” So, this assumption is correct.
Assumption (2) is correct. The passage mentions, “The basic argument is that these are a waste of
resources and place a burden on already stressed fiscal resources.” It implies a negative impact on the
economy. The passage further mentions, “MGNREGA for instance has been another scheme which has
been a lifeline for many during the pandemic and earlier.” It implies a positive impact on the society. So,
this assumption is correct.
Assumption (3) is correct. The passage mentions, “welfare schemes such as free or subsidized rations
under the Public Distribution System (PDS), cooked meals under the mid-day meal scheme,
supplementary nutrition through anganwadis, and work provided through the Mahatma Gandhi National
Rural Employment Guarantee Act (MGNREGA). MGNREGA for instance has been another scheme which
has been a lifeline for many during the pandemic and earlier. At a time when there are few employment
opportunities, working under MGNREGA can guarantee some assured wages”. It implies, welfare
schemes protect basic human rights to food and work, which fulfil the right to life with dignity and further
contribute to human development. So, this assumption is correct.
25.(b)
Assumption 1 is invalid. The given assumption is incorrect because the following line “Violations of the
right to education may occur through the direct action of States parties (the act of commission) or
through their failure to take steps required by law (the act of omission)”, illustrates that violations can
happen through an act of omission as well, which means that even after integrating the right in national
laws there is no surety that violations will not happen. So, to assume that integration will end violations is
not correct.
Assumption 2 is valid. As per the passage, “Violations of the right to education may occur through the
direct action of States parties (the act of commission) or through their failure to take steps required by
law (the act of omission)”, the state through an act of commission or omission can violate the right to
education. So, this assumption is correct.
26.(d)
We have to find the day on 1st April, 2002.
1st April, 2002 = (2000 years + time period from 1.1.2002 to 1.4.2002)
1600 years » 0 odd days
400 years » 0 odd days
2001 » 1 odd day
In 2002, Jan + Feb + March = (31 + 28 + 31) = 90 days ≡ 6 odd days.
So, Total number of odd days = (0 + 0 + 1 + 6) = 7 ≡ 0
On April 1, 2002 it was Monday.
So, in the month of April 2002, Tuesday fell on 2nd, 9th, 16th, 23rd & 30th.
27.(c)
The given figure is:
.
6 www.visionias.in ©Vision IAS
In rectangle EFHG, minimum 4 straight lines are there (i.e. EF, FH, GH, and EG)
Similarly, in rectangle CDJI, there are minimum 4 straight lines.
YT and MN are 2 straight lines.
GO, GM, HP, HN are 4 straight lines.
In triangle QKL, there are minimum 4 straight lines (i.e. QK, QR, QL and KL).
In quadrilateral ACDB, minimum 5 straight lines are there (i.e. AC, CY, YD, DB and AB).
Hence, minimum number of straight lines in the given figure = 4 + 4 + 2 + 4 + 4 + 5 = 23
Hence, option (c) is the correct answer.
28.(b)
Let the number of pants purchased be x and the number of skirts purchased be y.
It is given that, the number of skirts is two less than twice the number of pants purchased.
So, y = 2x – 2
or 2x – y = 2 ……..(i)
Also, the number of skirts is four less than four times the number of pants purchased
So, y = 4x – 4
or 4x – y = 4 ……… (ii)
Subtracting equation (i) from equation (ii), we get:
x = 1
Putting this value of x in equation (i), we get:
y = 0
Therefore, Sheetal bought 1 pant and 0 skirt.
Hence, option (b) is the correct answer.
29.(b)
R1 : 16 18 22 26
R2 : 8 9 10 13
R3 : 20 16 10 11
R4 : 28 25 22 ?
The pattern is: R1 + R3 – R2 = R4
16 + 20 – 8 = 28
18 + 16 – 9 = 25
22 + 10 – 10 = 22
26 + 11 – 13 = 24
Hence, option (b) is the correct answer.
30.(d)
Time is 2:20.
Position of the hands:
* Hour hand at 2 (nearly).
* Minute hand at 4
Angle between 2 and 4 is 60 degrees.
Angular distance travelled by the hour hand in 20 minutes is 10 degrees, since it turns through ½ degrees
in a minute.
Therefore, angle between the hands at 2:20 = 60 degrees - 10 degrees = 50 degrees
31.(d)
At 11 O'clock, the hour hand is 5 spaces apart from the minute hand.
During the next 60 minutes, i.e. between 11' O clock and 12' O clock the minute hand will move further
away from the hour hand. It will eventually meet the hour hand exactly at 12 O'clock.
32.(b)
After every 400 years, the calendar repeats.
Thus, if 27th February, 2003 was a Thursday, then on 27th February, 1603 it must have been a Thursday
too.
.
7 www.visionias.in ©Vision IAS
33.(b)
The clock loses 1% week-time during the first week.
In a day there are 24 hours and in a week there are 7 days. Therefore, there are 7 × 24 = 168 hours in a
week.
If the clock loses 1% time during the first week, then it will show a time which is 1% of 168 hours less
than 12 Noon at the end of the first week = 1.68 hours less.
Subsequently, the clock gains 2% during the next week. The second week has 168 hours and the clock
gains 2% time = 2% of 168 hours = 3.36 hours more than the actual time.
As it lost 1.68 hours during the first week and then gained 3.36 hours during the next week, the net result
will be a -1.68 + 3.36 = 1.68 hours net gain in time.
So the clock will show a time which is 1.68 hours more than 12 noon two weeks from the time it was set
right.
1.68 hours = 1 hour and 40.8 minutes, i.e. 1 : 40 P.M.
34.(d)
Explanation for Questions 35 to 38:
LR was the third movie to be screened on screen 4. PN was screened on two other screens prior to screen
3, and was screened on 1 immediately after it.
was screened on 3.
SCREEN 1 SCREEN 2 SCREEN 3 SCREEN 4 SCREEN 5
DAY 1
DAY 2
DAY 3 PN LR
DAY 4 PN
DAY 5
In all the screens, except screen 2, IR was screened just before IN. IN was released on screen 3 on day 2.
SCREEN 1 SCREEN 2 SCREEN 3 SCREEN 4 SCREEN 5
DAY 1 IR
DAY 2 IR IN
DAY 3 IN PN LR IR
DAY 4 PN IR IN
DAY 5 IN
Now, on screen 4 LR was screened two days after PN.
SCREEN 1 SCREEN 2 SCREEN 3 SCREEN 4 SCREEN 5
DAY 1 IR PN
DAY 2 IR IN ER
DAY 3 IN PN LR IR
DAY 4 PN IR IN
DAY 5 IN
On day 1, PN and ER were released on two consecutive screen numbers.
SCREEN 1 SCREEN 2 SCREEN 3 SCREEN 4 SCREEN 5
DAY 1 IR PN ER
DAY 2 IR IN ER
DAY 3 IN PN LR IR
DAY 4 PN IR IN
DAY 5 IN
Now, let’s fill up the gaps in screen 1.
SCREEN 1 SCREEN 2 SCREEN 3 SCREEN 4 SCREEN 5
DAY 1 LR IR PN ER
DAY 2 IR IN ER
DAY 3 IN PN LR IR
DAY 4 PN IR IN
DAY 5 ER IN
.
8 www.visionias.in ©Vision IAS
Now, let’s fill up the gaps in screen 3 and day 4.
SCREEN 1 SCREEN 2 SCREEN 3 SCREEN 4 SCREEN 5
DAY 1 LR IR PN ER
DAY 2 IR IN ER
DAY 3 IN PN LR IR
DAY 4 PN LR ER IR IN
DAY 5 ER LR IN
Now, let’s fill up the gaps in screen 5 and day 5.
SCREEN 1 SCREEN 2 SCREEN 3 SCREEN 4 SCREEN 5
DAY 1 LR IR PN ER
DAY 2 IR IN ER LR
DAY 3 IN PN LR IR
DAY 4 PN LR ER IR IN
DAY 5 ER IR LR IN PN
Now, let’s fill up the gaps in screen 2.
SCREEN 1 SCREEN 2 SCREEN 3 SCREEN 4 SCREEN 5
DAY 1 LR IN IR PN ER
DAY 2 IR PN IN ER LR
DAY 3 IN ER PN LR IR
DAY 4 PN LR ER IR IN
DAY 5 ER IR LR IN PN
35.(b)
36.(a)
37.(c)
38.(b)
39.(a)
Statements 1 and 3 are correct. The lines “Drought will make water more expensive, which will likely
affect the cost of raw materials and production. Climate volatility may force companies to deal with
uncertainty in the price of resources for production, energy transport and insurance. And some products
could become obsolete or lose their markets, such as equipment related to coal mining or skiing in an
area that no longer has snow” reflect that options 1 and 3 are disadvantages of climate change.
Statements 2 and 4 are incorrect. The lines “Whether in the U.S. or abroad, new regulations such as
carbon pricing and subsidies that favour a competitor may affect a business’s bottom line. A company’s
reputation could also suffer if it’s seen as doing something that hurts the environment”, this shows that
carbon pricing is important for dealing with issues of climate change and also, if the companies are forced
to think about the environment, then also, it is good for dealing with climate change. So, these are not
disadvantages but benefits.
40.(a)
Option (a) is correct. The lines “Drought will make water more expensive, which will likely affect the
cost of raw materials and production. Climate volatility may force companies to deal with uncertainty in
the price of resources for production, energy transport and insurance. And some products could become
obsolete or lose their markets, such as equipment related to coal mining or skiing in an area that no
longer has snow. Whether in the U.S. or abroad, new regulations such as carbon pricing and subsidies
that favour a competitor may affect a business’s bottom line.”, illustrate the changes or impact of climate
change on companies and the production of goods. Therefore, this option is the best crux of the passage.
Option (b) is incorrect. The passage discusses the companies but to say that companies which do not
follow climate governance will face a boycott of their products will not be correct. So, this option is
beyond the scope of the passage.
Option (c) is incorrect. This option in general seems to be correct but it is not based on the information
given in the passage. The passage mainly focuses on the challenges of climate change from the
.
9 www.visionias.in ©Vision IAS
perspective of companies (not nations) and what impact will it have on their production methods. So, this
is also not correct.
Option (d) is incorrect. There is no discussion about the significance of sustainable development in
coping with climate change. So, this option is beyond the scope of the passage.
41.(d)
Option (a) is incorrect. The author is not pessimistic about sufferings in this world. Towards the
conclusion, the author says people can be liberated from sufferings when they understand the
impermanent nature of all their feelings, and stop craving for them. Thus, despite feelings being never-
ending and ephemeral, sufferings can end.
Option (b) is incorrect. In the very beginning the author says, the root of suffering is neither the feeling of
pain nor of sadness nor even of meaninglessness. It is nowhere said that these feelings ONLY in
combinations of two or three lead to sufferings. Rather, according to the author, real root of suffering is
never-ending and pointless pursuit of ephemeral (short-lived) feelings.
Option (c) is also incorrect. This statement on its own might be true. However, it cannot be inferred from
this passage, as the author does not make any reference to Buddha.
42.(d)
Angle between 3 and 8 on the clock = 240 - 90 = 150 degrees.
The angle travelled by the hour hand from 3 = (40/60) x 30 = 20 degrees.
Therefore, the net angle between the hour hand and the minute hand = 150 - 20 = 130 degrees.
43.(c)
44.(b)
Explanation for Questions 45 to 48:
Minimum possible number of times the operations could have been performed are as follows:
6:00 AM to 8:00 AM
Operation I: Net addition of peaches = 113 – 103 = 10
Operation II: Number of peaches redistributed = [Increase in the number of peaches in boxes A, B and C –
Net addition of peaches] = 18 – 10 = 8
Operation III: 0
Total = 18
8:00 AM to 10:00 AM
Operation I: Net addition of peaches = 125 – 113 = 12
Operation II: Number of peaches redistributed = [Increase in the number of peaches in boxes A, B and C –
Net addition of peaches] = 17 – 12 = 5
Operation III: 0
Total = 17
10:00 AM to 12:00 PM
Operation I: Net addition of peaches = 0
Operation II: Number of peaches redistributed = 1 (say from D to B)
Operation III: Peaches thrown away = (29 - 26) + (40 - 39) = 4
Total = 5
12:00 PM to 2:00 PM
Operation I: Net addition of peaches = 134 – 121 = 13
Operation II: Number of peaches redistributed = [Increase in the number of peaches in boxes A and D –
Net addition of peaches] = 23 - 13 = 10
Operation III: 0
Total = 23
So, total number of operations performed by Kumar = 18 + 17 + 5 + 23 = 63.
45.(c)
46.(b)
.
10 www.visionias.in ©Vision IAS
47.(c)
From 6:00 AM to 8:00 AM: 8 – (25 – 23) – (31 – 27) = 2
From 8:00 AM to 10:00 AM : 5 – (29 – 25) = 1
From 10:00 AM to 12:00 PM: 0
From 12:00 PM to 2:00 PM: 0
Therefore, the required answer = 2 + 1 = 3.
48.(a)
Required number = 8 + 5 + 1 + 10 = 24.
49.(b)
Assumption 1 is invalid. The given statement is extreme. As per the lines “This approach includes
educating people about dietary changes and helping them achieve optimum levels of nutrition. An
important part of this is educating children at schools and people in shared spaces such as workplaces.
The main aim of this approach is preventive”, community nutrition is preventive but to assume that it is
not possible to overcome malnutrition by obesity would not be correct because there is no such relation
being discussed in the passage.
Assumption 2 is invalid. The lines “We currently have no significant programmes that target
malnutrition affected by obesity”, merely show that there are no significant programmes to tackle
malnutrition by obesity, but to assume that the absence of programmes is the primary cause of
malnutrition would not be correct. Also, the primary cause of malnutrition is not discussed in the passage.
So, this assumption is not correct.
Assumption 3 is valid. The given statement is about replacing packaged and high-salt food with water-
rich live foods because the lines “With the rise of packaged food, food with high salt and sugar content in
food obesity-related malnutrition is on the rise. India is now facing a double whammy of malnutrition,
with the younger population being affected by undernutrition and the older population affected by
malnutrition related to obesity”, show that packaged or high-salt foods are the cause of malnutrition.
Therefore, consuming fresh and nutrition rich food will help fight malnutrition.
50.(b)
Option (a) is incorrect. The given option states that community nutrition is the best approach. However,
such a conclusion can only be made when two approaches are compared which is not done in the passage.
So, to conclude that the community nutrition approach is the best would not be correct.
Option (b) is correct. The passage is mainly based on the theme of the use of a community nutrition
approach to fight malnutrition by obesity. The lines “We currently have no significant programmes that
target malnutrition affected by obesity. As a country, we could take the community nutrition approach
where nutrition is improved at the community level”, show that India needs to focus on handling
malnutrition by obesity and the community nutrition approach is a promising way to do it.
Option (c) is incorrect. The option talks about a change of government approach in fighting malnutrition
by obesity. However, the main theme of the passage is not about issues with government programmes but
the use of a community nutrition approach in fighting malnutrition by obesity. So, this option is not the
best message of the author.
Option (d) is incorrect. The given option is beyond the scope of the passage because there is no mention
of companies producing packaged foods, and also what needs to be done on their behalf. There is no
discussion about the regulation of such companies. So, this is not the best message of the author.
51.(b)
Inference 1 is incorrect. The given statement is partially correct. The assertion about being dangerous to
the environment is correct. However, to say that it has some potential benefits is not correct because it is
not based on any information in the passage. So, on whole, the inference is not correct.
Inference 2 is correct. The lines “Invasive mammalian predators are therefore important drivers of
irreversible loss of phylogenetic diversity worldwide. That most impacted species are insular indicates
that management of invasive predators on islands should be a global conservation priority”, illustrating
that the island being an insular habitat contains species which are most impacted by invasive predators.
Therefore, island species are most vulnerable and need conservation on a priority basis.
.
11 www.visionias.in ©Vision IAS
52.(d)
Option (a) is incorrect. The given option is beyond the scope of the passage because there is no relation
drawn between the survival of the human race and the conservation of biodiversity which is being
discussed in the passage.
Option (b) is incorrect. The given option is not correct because there is no discussion about the funding
of islands for conserving biodiversity there or fighting invasive species. So, this option is beyond the
scope of the passage.
Option (c) is incorrect. This given statement is not based on the information in the passage. Also, it is
extreme because to conclude that invasive species can grow anywhere would not be correct without any
supporting information. Therefore, this option is beyond the scope of the passage.
Option (d) is correct. The lines “Invasive predators are implicated in 87 bird, 45 mammals, and 10
reptile species extinctions - 58% of these groups’ contemporary extinctions worldwide. These figures are
likely underestimated because 23 critically endangered species that were assessed are classed as
“possibly extinct”, show that invasive predators are the cause of extinction for many species. Extinction
of any species from the ecosystem alters its species diversity. Therefore, the given assertion in the
statement is correct and it is the best crux of the passage.
53.(d)
Let the number of keepers be x.
Then, total number of feet = 2 × 50 + 4 × 45 + 4 × 8 + 2x = 2x + 312.
Now, (2x + 312) = (103 + x) + 224
or x = 15.
54.(d)
The seven pieces consist of 6 smaller equal pieces and one half cake piece.
Weight of each small piece = 20 g.
So, total weight of the cake = [2 × (20 × 6)]g = 240 g.
55.(d)
Total relative angular distance to be covered by minute hand = 7 ×30° = 210°
Minute hand gains 11/2° over hour hand in 1 minute
Time required to make 210° gain = (2/11)×210 = 420/11 = 38(2/11) min. past 1
56.(a)
57.(a)
See the third row. First box has 2 circles & 0 triangles, 2nd
has 1 circle (decrease by 1) & 1 triangle
(increase by 1). So, 3rd
box must have 0 circles ad 2 triangles. Similar pattern is followed in all rows.
You can also solve this by considering columns instead of rows.
58.(b)
It is given that 28th August, 1946 was a Wednesday. From 28th August, 1946 to 28th August 1961, we
have 4 leap years and 11 normal years.
So, the number of odd days = 11×1 + 4×2 = 19 = 5
Now we have to move from 28th August to 31st August, i.e. 3 more odd days.
So, total number of odd days = 5 + 3 = 8 = 1
So, 31st August 1961 would be Wednesday + 1 = Thursday
59.(b)
Palmyra < Date palm; Date palm < Queen palm; Queen palm > Caryota
After combining first two statements, we get: Palmyra < Date palm < Queen palm and Caryota < Queen
palm
[Position of Caryota is not certain but it is lesser than Queen palm for sure, so it can be at any place below
Queen palm]
Of the given options, only option (B), i.e. Palmyra < Queen palm, is definitely always true.
.
12 www.visionias.in ©Vision IAS
60.(c)
From condition 4, we can get the exact position of class IV (to which Hindi teacher went) and the exact
position of the class of Science teacher. Condition 5 says that there are four classes to the left of class l.
So, one class will be to the right of class I. It is also given that class IV and lll are not adjacent to each
other. Hence we get:
As per condition 3, English teacher went to the class to the immediate left of class I. As per condition 2,
Math teacher went to class III which is adjacent to class II. Thus, class III cannot lie to the right of class I.
Hence, class III will lie second to the left of class I.
Now, as per condition 1, Drawing teacher went to the class to the immediate left of class V and to the
immediate right of class ll. Therefore, class V will lie on rightmost side and Drawing teacher will take
class I. Hence, final arrangement will be:
We can see that, Math teacher went to class III.
61.(a)
Science teacher went to class VI.
62.(c)
Class I is to the immediate right of class II, and is taken by Drawing teacher.
63.(b)
Assumption 1 is invalid. The given assumption is not correct because the lines “Currently, there are U.S.
public programs that promote renewable energy options and recommendations in conserving fuel and
energy for farms, efficient use of fertilizers and pesticides, soil conservation measures, and energy-
efficient methods of growing and transporting foods”, talk about replacing fossil fuels with renewable
energy. It means that agriculture in the U.S is not completely dependent on fossil fuels. Therefore, to
assume that its future is bleak would not be correct.
Assumption 2 is valid. The lines “Currently, there are U.S. public programs that promote renewable
energy options and recommendations in conserving fuel and energy for farms, efficient use of fertilizers
and pesticides, soil conservation measures, and energy-efficient methods of growing and transporting
foods, but those programs will need high levels of participation to curb both climate impacts as well as
address air and water quality issues associated with traditional agricultural practices”, show that along
with replacing fossil fuels with renewable energy, it is important to ensure high levels of participation to
curb climate change and address air and water quality issues of agriculture. So, it is correct to assume that
only fossil-fuel replacement may not address the air and water pollution caused due to agriculture.
64.(a)
All Statements are correct. The line “Direct energy use in agriculture is primarily from fossil fuels used
to operate automobiles and machinery for preparing fields, planting, and harvesting crops, applying
chemicals, and transporting inputs and outputs to and from the market”, illustrates that all the given
options are the possible uses of energy in agriculture.
65.(d)
Statement 1 is correct. It is mentioned in the passage that the JKEAC had looked at the mining blocks in
the most recent October 2021 Google Image. It was discovered that extensive unlawful mining has
.
13 www.visionias.in ©Vision IAS
significantly reduced the availability of minerals. Therefore, it can be concluded that the government is
getting help from technology for environmental protection. Hence, it is a correct statement.
Statement 2 is correct. It is mentioned in the passage that in Kashmir and Himachal Pradesh, Kuhls are
conventional irrigation systems that are run by the local community. Further, it says there are also
irrigation Kuhls in the mining area and mining is detrimental to the flow of water. Hence, it is a correct
statement.
Statement 3 is correct. The passage says that the Hokersar wetland is fed by the Shaliganga Nallah, and
mining activity may harm the water's natural flow and quality as it enters Hokersar. Hence, it is a correct
statement.
66.(d)
Statement 1 is correct as the passage mentions, “The rapidly snowballing loss in diversity is largely
triggered and fuelled by new age emergencies and interventions, like climate change, change in
atmospheric composition and the spread of invasive alien species.” Hence statement 1 is correct.
Statement 2 is correct as the passage mentions, “The rapidly snowballing loss in diversity is largely
triggered and fuelled by new age emergencies and interventions, like climate change, change in
atmospheric composition and the spread of invasive alien species.” and also warming of temperature
above pre-industrial levels that has impacted agriculture and contributed to the migration of species.
Hence statement 2 is correct.
Statement 3 is correct as the passage mentions, “In just the last three centuries, global forest areas have
shrunk by 40 per cent. Every year, to meet the timber needs from natural sources, the Earth is stripped of
100 million trees.”, which clearly indicates deforestation. Its impact has been the risk of faster extinction
in the plant kingdom. Hence, statement 3 is correct too.
67.(c)
Originally, let number of women = x. Then number of men = 2x.
So, in city Y:
(2x – 10) = (x + 5)
or x = 15.
So, total number of passengers in the beginning = (x + 2x) = 3x = 45.
68.(d)
Given equation: 5# × 3#0 ÷ #0 – 3#3 + #3 = 3#
Let us use the options:
From option (a), # = 4
54 × 340 ÷ 40 – 343 + 43 ≠ 34 (not satisfied)
From option (b), # = 5
55 × 350 ÷ 50 – 353 + 53 ≠ 35 (not satisfied)
From option (c), # = 8
58 × 380 ÷ 80 – 383 + 83 ≠ 38 (not satisfied)
From option (d), # = 6
56 × 360 ÷ 60 – 363 + 63 = 36 (satisfied)
Hence, option (d) is the correct answer.
69.(b)
x weeks x days = (7x + x) days = 8x days.
70.(c)
If yesterday were tomorrow, then today would be Friday. So, day before yesterday was Friday.
So, today is a Sunday.
Explanation for Questions 71 to 74:
Consider members of groups C and E. From I and II, R and T are not Y’s family members. X and Y are
members of group E. So, X is not from Y’s family. So, group C has member V from Y’s family.
Q and Y & Z are members of group B. O and V & Z are members of group D. So, O and Q are not
members of Y or Z’s family. Also, R and Y are not from the same family. So, S must be from the family
.
14 www.visionias.in ©Vision IAS
of Y and V, and R must be from the family of Z (inferred from data in group A). Also, from III and IV, Y,
V and S belong to either Mishra's or Malhotra's.
Group A O Q R (family 2) S (family 1)
Group B Q W Y (family 1) Z (family 2)
Group C R (family 2) T V (family 1) X
Group D O U V (family 1) Z (family 2)
Group E P U X Y (family 1)
Now consider groups A and D. Neither Q nor Z are from same family as O. So, among members of group
B, Q, Z and Y are not from Ahuja family. So, W has to be from Ahuja family.
R and V are not Ahuja family members. Also, T is not from the Ahuja’s. So, X must be from the Ahuja’s
(inferred from group C). Thus, Ahuja family: O, W and X.
Group A O (Ahuja) Q R (family 2) S (family 1)
Group B Q W (Ahuja) Y (family 1) Z (family 2)
Group C R (family 2) T V (family 1) X (Ahuja)
Group D O (Ahuja) U V (family 1) Z (family 2)
Group E P U X (Ahuja) Y (family 1)
Consider remaining members of groups A, B, C, and D. It can be concluded that R, Z and P are members
of the same family, while Q, T and U are from one family too.
Group A O (Ahuja) Q (family 3) R (family 2) S (family 1)
Group B Q (family 3) W (Ahuja) Y (family 1) Z (family 2)
Group C R (family 2) T (family 3) V (family 1) X (Ahuja)
Group D O (Ahuja) U (family 3) V (family 1) Z (family 2)
Group E P (family 2) U (family 3) X (Ahuja) Y (family 1)
Now, from IV, Sharma family is family 2: R, Z and P. Q, T and U belong to family 3, i.e., either
Malhotra's or Mishra's.
Group A O (Ahuja) Q (family 3) R (Sharma) S (family 1)
Group B Q (family 3) W (Ahuja) Y (family 1) Z (Sharma)
Group C R (Sharma) T (family 3) V (family 1) X (Ahuja)
Group D O (Ahuja) U (family 3) V (family 1) Z (Sharma)
Group E P (Sharma) U (family 3) X (Ahuja) Y (family 1)
71.(b)
X is from the same family as W.
72.(c)
Both Z and P are from the Sharma family.
73.(d)
74.(d)
We can determine that O, W and X are from Ahuja family, and P, R and Z are from Sharma family.
75.(c)
When Rahul was born:
His brother’s age = 6 years,
His father’s age = (6 + 32) = 38 years.
His mother’s age = (38 – 3) years = 35 years.
His sister’s age = (35 – 25) years = 10 years.
76.(d)
Statement 1 is incorrect as the passage mentions “Especially in countries that score high on the World
Happiness Index (WHI), there is a strong link between the feeling of needing to be happy and the extent to
which people actually experience feelings such as sadness, gloom, fatigue or anxiety." It does not mean
.
15 www.visionias.in ©Vision IAS
that the link between needing to be happy and negative emotions is weak in countries that score low on
WHI. There is insufficient information to arrive at the conclusion. Hence statement 1 is incorrect.
Statement 2 is incorrect as the passage mentions that social media, in self-help books and advertisements
create an image of "happiness norm". Society has created an image of what it means to be happy.
However, it does not imply that it actually leads to happiness in life. Hence statement 2 is incorrect.
77.(c)
Option (a) is incorrect as the passage mentions “autonomy goes beyond individualism and self-
expression and involves the exercise of duties, obligations, and respect for other moral agents and their
own rights." The passage does not mention whether the patient has the right to treatment. Hence statement
1 is incorrect.
Option (b) is incorrect as although the passage emphasises the autonomy of the patient, whether such
right has to remain inviolable at all the times is not clear from the passage. Further, the main message of
the passage is balance between patient's rights and doctor's rights. Hence statement 2 is incorrect.
Option (c) is correct as the passage mentions "Ideas of autonomy have not only been related to the
individual self-expression of the patient; indeed, they have been used to validate arguments for clinical
independence and discretion. O’Neill has argued, following Kant, that autonomy goes beyond
individualism and self-expression and involves the exercise of duties, obligations, and respect for other
moral agents and their own rights." Therefore, the emphasis is on balance between patient's rights and
doctor's rights. Hence statement 3 is correct.
Option (d) is incorrect as although the statement is factually true in this context, it does not form the
central argument of the passage, as stated above. Hence statement 4 is incorrect.
78.(a)
Statement 1 is correct. The passage says that the new visa system was put into place in an effort to
"enhance the quality of life and make the experience of living, working, and investing in the UAE a happy
and cheerful one." Therefore, it can be inferred that the new visa system seeks to increase investment in
UAE.
Statement 2 is incorrect. As mentioned in the passage, with the introduction of a flexible, multiple-entry
tourist visa with a five-year validity period, they can stay in the UAE for up to 90 days nonstop. Hence, it
is not a correct statement.
79.(b)
Statement 1 is incorrect. According to the passage, a census or survey is the best approach to determine
whether the majority of Indian families are still considered "joint families”, and if one wants to compare
the situation of women in nuclear and combined families, participant observation, case studies, and
interviews may all be useful techniques. Hence it is not a correct statement.
Statement 2 is correct. As mentioned in the passage there are numerous methodologies in sociology
given the multiplicity of perspectives and truths. The path to social truth is not one and the same. Hence, it
is a correct statement.
80.(b)
Option (a) is incorrect. According to the passage, it is a partially incorrect statement. It is only a part of
the central argument which says that multiple truths exist in sociology, simultaneously. Hence, it is not a
correct statement.
Option (b) is correct. It is the most important implication of the passage. The passage says that there are
numerous methodologies in sociology given the multiplicity of perspectives and truths. The path to social
truth is not one and the same. The passage revolves around this central argument. Hence, it is the correct
statement.
Option (c) is incorrect. It is not mentioned anywhere in the passage. Hence, it is not a correct statement.
Option (d) is incorrect. In the sociological study, quantitative and qualitative methods are used. But
among the two which one is more useful and used more is not mentioned. Hence, it is not a correct
statement.
Copyright © by Vision IAS
All rights are reserved. No part of this document may be reproduced, stored in a retrieval system or transmitted
in any form or by any means, electronic, mechanical, photocopying, recording or otherwise, without prior
permission of Vision IAS.

More Related Content

Similar to 1_5071503950121796163.pdf

Indian Administrative Service(IAS) CSAT PAPER I | STUDY MATERIALS | NOTES SOL...
Indian Administrative Service(IAS) CSAT PAPER I | STUDY MATERIALS | NOTES SOL...Indian Administrative Service(IAS) CSAT PAPER I | STUDY MATERIALS | NOTES SOL...
Indian Administrative Service(IAS) CSAT PAPER I | STUDY MATERIALS | NOTES SOL...
SOURAV DAS
 
Ias csat paper 2 -- 2018 COMPLETE SOLUTION SOURAV SIR'S CLASSES 9836793076
Ias csat paper 2   --   2018 COMPLETE SOLUTION SOURAV SIR'S CLASSES  9836793076Ias csat paper 2   --   2018 COMPLETE SOLUTION SOURAV SIR'S CLASSES  9836793076
Ias csat paper 2 -- 2018 COMPLETE SOLUTION SOURAV SIR'S CLASSES 9836793076
SOURAV DAS
 
gymnastic instructor
gymnastic instructorgymnastic instructor
gymnastic instructoranita
 
Homework #1SOCY 3115Spring 20Read the Syllabus and FAQ on ho.docx
Homework #1SOCY 3115Spring 20Read the Syllabus and FAQ on ho.docxHomework #1SOCY 3115Spring 20Read the Syllabus and FAQ on ho.docx
Homework #1SOCY 3115Spring 20Read the Syllabus and FAQ on ho.docx
pooleavelina
 
How to download movies from odiamedia.in
How to download movies from odiamedia.inHow to download movies from odiamedia.in
How to download movies from odiamedia.in
Openjobalert
 
BMS
BMSBMS
Education In Chile Essay
Education In Chile EssayEducation In Chile Essay
Education In Chile Essay
Katy Shaw
 
Potential questions and study guide for exam 1 Any 8 of.docx
Potential questions and study guide for exam 1  Any 8 of.docxPotential questions and study guide for exam 1  Any 8 of.docx
Potential questions and study guide for exam 1 Any 8 of.docx
ChantellPantoja184
 
BUSI 620 Inspiring Innovation/tutorialrank.com
 BUSI 620 Inspiring Innovation/tutorialrank.com BUSI 620 Inspiring Innovation/tutorialrank.com
BUSI 620 Inspiring Innovation/tutorialrank.com
jonhson108
 
Busi 620 Enhance teaching / snaptutorial.com
Busi 620  Enhance teaching / snaptutorial.comBusi 620  Enhance teaching / snaptutorial.com
Busi 620 Enhance teaching / snaptutorial.com
HarrisGeorg50
 
XAT 2014 Previous Year Question Paper
XAT 2014 Previous Year Question PaperXAT 2014 Previous Year Question Paper
XAT 2014 Previous Year Question Paper
Eneutron
 
Development final
Development   finalDevelopment   final
Development final
Harjas Gulati
 
1. Planet Earth is our only home for the time being. As global.docx
1. Planet Earth is our only home for the time being. As global.docx1. Planet Earth is our only home for the time being. As global.docx
1. Planet Earth is our only home for the time being. As global.docx
braycarissa250
 
federal-government-scholarship-past-questions.pdf
federal-government-scholarship-past-questions.pdffederal-government-scholarship-past-questions.pdf
federal-government-scholarship-past-questions.pdf
ZakariyahAbdulaleem
 
Quiz 1.pdf
Quiz 1.pdfQuiz 1.pdf
Quiz 1.pdf
Shan69025
 
Here is an example of a question along with a possible answe.docx
Here is an example of a question along with a possible answe.docxHere is an example of a question along with a possible answe.docx
Here is an example of a question along with a possible answe.docx
pooleavelina
 
UPSC_CSAT_previous_year_question_paper_Prelims_GS-II_2017.pdf
UPSC_CSAT_previous_year_question_paper_Prelims_GS-II_2017.pdfUPSC_CSAT_previous_year_question_paper_Prelims_GS-II_2017.pdf
UPSC_CSAT_previous_year_question_paper_Prelims_GS-II_2017.pdf
ungistcsat
 
1Answer the following questions1. Jackson even-numbered C.docx
1Answer the following questions1. Jackson even-numbered C.docx1Answer the following questions1. Jackson even-numbered C.docx
1Answer the following questions1. Jackson even-numbered C.docx
hyacinthshackley2629
 
ECON 301 Midterm IClosed book and notes. Calculators are a.docx
ECON 301 Midterm IClosed book and notes. Calculators are a.docxECON 301 Midterm IClosed book and notes. Calculators are a.docx
ECON 301 Midterm IClosed book and notes. Calculators are a.docx
budabrooks46239
 
2.The objective of this question is to help you •understand the.docx
2.The objective of this question is to help you •understand the.docx2.The objective of this question is to help you •understand the.docx
2.The objective of this question is to help you •understand the.docx
domenicacullison
 

Similar to 1_5071503950121796163.pdf (20)

Indian Administrative Service(IAS) CSAT PAPER I | STUDY MATERIALS | NOTES SOL...
Indian Administrative Service(IAS) CSAT PAPER I | STUDY MATERIALS | NOTES SOL...Indian Administrative Service(IAS) CSAT PAPER I | STUDY MATERIALS | NOTES SOL...
Indian Administrative Service(IAS) CSAT PAPER I | STUDY MATERIALS | NOTES SOL...
 
Ias csat paper 2 -- 2018 COMPLETE SOLUTION SOURAV SIR'S CLASSES 9836793076
Ias csat paper 2   --   2018 COMPLETE SOLUTION SOURAV SIR'S CLASSES  9836793076Ias csat paper 2   --   2018 COMPLETE SOLUTION SOURAV SIR'S CLASSES  9836793076
Ias csat paper 2 -- 2018 COMPLETE SOLUTION SOURAV SIR'S CLASSES 9836793076
 
gymnastic instructor
gymnastic instructorgymnastic instructor
gymnastic instructor
 
Homework #1SOCY 3115Spring 20Read the Syllabus and FAQ on ho.docx
Homework #1SOCY 3115Spring 20Read the Syllabus and FAQ on ho.docxHomework #1SOCY 3115Spring 20Read the Syllabus and FAQ on ho.docx
Homework #1SOCY 3115Spring 20Read the Syllabus and FAQ on ho.docx
 
How to download movies from odiamedia.in
How to download movies from odiamedia.inHow to download movies from odiamedia.in
How to download movies from odiamedia.in
 
BMS
BMSBMS
BMS
 
Education In Chile Essay
Education In Chile EssayEducation In Chile Essay
Education In Chile Essay
 
Potential questions and study guide for exam 1 Any 8 of.docx
Potential questions and study guide for exam 1  Any 8 of.docxPotential questions and study guide for exam 1  Any 8 of.docx
Potential questions and study guide for exam 1 Any 8 of.docx
 
BUSI 620 Inspiring Innovation/tutorialrank.com
 BUSI 620 Inspiring Innovation/tutorialrank.com BUSI 620 Inspiring Innovation/tutorialrank.com
BUSI 620 Inspiring Innovation/tutorialrank.com
 
Busi 620 Enhance teaching / snaptutorial.com
Busi 620  Enhance teaching / snaptutorial.comBusi 620  Enhance teaching / snaptutorial.com
Busi 620 Enhance teaching / snaptutorial.com
 
XAT 2014 Previous Year Question Paper
XAT 2014 Previous Year Question PaperXAT 2014 Previous Year Question Paper
XAT 2014 Previous Year Question Paper
 
Development final
Development   finalDevelopment   final
Development final
 
1. Planet Earth is our only home for the time being. As global.docx
1. Planet Earth is our only home for the time being. As global.docx1. Planet Earth is our only home for the time being. As global.docx
1. Planet Earth is our only home for the time being. As global.docx
 
federal-government-scholarship-past-questions.pdf
federal-government-scholarship-past-questions.pdffederal-government-scholarship-past-questions.pdf
federal-government-scholarship-past-questions.pdf
 
Quiz 1.pdf
Quiz 1.pdfQuiz 1.pdf
Quiz 1.pdf
 
Here is an example of a question along with a possible answe.docx
Here is an example of a question along with a possible answe.docxHere is an example of a question along with a possible answe.docx
Here is an example of a question along with a possible answe.docx
 
UPSC_CSAT_previous_year_question_paper_Prelims_GS-II_2017.pdf
UPSC_CSAT_previous_year_question_paper_Prelims_GS-II_2017.pdfUPSC_CSAT_previous_year_question_paper_Prelims_GS-II_2017.pdf
UPSC_CSAT_previous_year_question_paper_Prelims_GS-II_2017.pdf
 
1Answer the following questions1. Jackson even-numbered C.docx
1Answer the following questions1. Jackson even-numbered C.docx1Answer the following questions1. Jackson even-numbered C.docx
1Answer the following questions1. Jackson even-numbered C.docx
 
ECON 301 Midterm IClosed book and notes. Calculators are a.docx
ECON 301 Midterm IClosed book and notes. Calculators are a.docxECON 301 Midterm IClosed book and notes. Calculators are a.docx
ECON 301 Midterm IClosed book and notes. Calculators are a.docx
 
2.The objective of this question is to help you •understand the.docx
2.The objective of this question is to help you •understand the.docx2.The objective of this question is to help you •understand the.docx
2.The objective of this question is to help you •understand the.docx
 

Recently uploaded

Thesis Statement for students diagnonsed withADHD.ppt
Thesis Statement for students diagnonsed withADHD.pptThesis Statement for students diagnonsed withADHD.ppt
Thesis Statement for students diagnonsed withADHD.ppt
EverAndrsGuerraGuerr
 
Digital Tools and AI for Teaching Learning and Research
Digital Tools and AI for Teaching Learning and ResearchDigital Tools and AI for Teaching Learning and Research
Digital Tools and AI for Teaching Learning and Research
Vikramjit Singh
 
S1-Introduction-Biopesticides in ICM.pptx
S1-Introduction-Biopesticides in ICM.pptxS1-Introduction-Biopesticides in ICM.pptx
S1-Introduction-Biopesticides in ICM.pptx
tarandeep35
 
Azure Interview Questions and Answers PDF By ScholarHat
Azure Interview Questions and Answers PDF By ScholarHatAzure Interview Questions and Answers PDF By ScholarHat
Azure Interview Questions and Answers PDF By ScholarHat
Scholarhat
 
The Challenger.pdf DNHS Official Publication
The Challenger.pdf DNHS Official PublicationThe Challenger.pdf DNHS Official Publication
The Challenger.pdf DNHS Official Publication
Delapenabediema
 
"Protectable subject matters, Protection in biotechnology, Protection of othe...
"Protectable subject matters, Protection in biotechnology, Protection of othe..."Protectable subject matters, Protection in biotechnology, Protection of othe...
"Protectable subject matters, Protection in biotechnology, Protection of othe...
SACHIN R KONDAGURI
 
BÀI TẬP BỔ TRỢ TIẾNG ANH GLOBAL SUCCESS LỚP 3 - CẢ NĂM (CÓ FILE NGHE VÀ ĐÁP Á...
BÀI TẬP BỔ TRỢ TIẾNG ANH GLOBAL SUCCESS LỚP 3 - CẢ NĂM (CÓ FILE NGHE VÀ ĐÁP Á...BÀI TẬP BỔ TRỢ TIẾNG ANH GLOBAL SUCCESS LỚP 3 - CẢ NĂM (CÓ FILE NGHE VÀ ĐÁP Á...
BÀI TẬP BỔ TRỢ TIẾNG ANH GLOBAL SUCCESS LỚP 3 - CẢ NĂM (CÓ FILE NGHE VÀ ĐÁP Á...
Nguyen Thanh Tu Collection
 
Group Presentation 2 Economics.Ariana Buscigliopptx
Group Presentation 2 Economics.Ariana BuscigliopptxGroup Presentation 2 Economics.Ariana Buscigliopptx
Group Presentation 2 Economics.Ariana Buscigliopptx
ArianaBusciglio
 
How to Make a Field invisible in Odoo 17
How to Make a Field invisible in Odoo 17How to Make a Field invisible in Odoo 17
How to Make a Field invisible in Odoo 17
Celine George
 
Supporting (UKRI) OA monographs at Salford.pptx
Supporting (UKRI) OA monographs at Salford.pptxSupporting (UKRI) OA monographs at Salford.pptx
Supporting (UKRI) OA monographs at Salford.pptx
Jisc
 
Unit 2- Research Aptitude (UGC NET Paper I).pdf
Unit 2- Research Aptitude (UGC NET Paper I).pdfUnit 2- Research Aptitude (UGC NET Paper I).pdf
Unit 2- Research Aptitude (UGC NET Paper I).pdf
Thiyagu K
 
Francesca Gottschalk - How can education support child empowerment.pptx
Francesca Gottschalk - How can education support child empowerment.pptxFrancesca Gottschalk - How can education support child empowerment.pptx
Francesca Gottschalk - How can education support child empowerment.pptx
EduSkills OECD
 
TESDA TM1 REVIEWER FOR NATIONAL ASSESSMENT WRITTEN AND ORAL QUESTIONS WITH A...
TESDA TM1 REVIEWER  FOR NATIONAL ASSESSMENT WRITTEN AND ORAL QUESTIONS WITH A...TESDA TM1 REVIEWER  FOR NATIONAL ASSESSMENT WRITTEN AND ORAL QUESTIONS WITH A...
TESDA TM1 REVIEWER FOR NATIONAL ASSESSMENT WRITTEN AND ORAL QUESTIONS WITH A...
EugeneSaldivar
 
A Survey of Techniques for Maximizing LLM Performance.pptx
A Survey of Techniques for Maximizing LLM Performance.pptxA Survey of Techniques for Maximizing LLM Performance.pptx
A Survey of Techniques for Maximizing LLM Performance.pptx
thanhdowork
 
Mule 4.6 & Java 17 Upgrade | MuleSoft Mysore Meetup #46
Mule 4.6 & Java 17 Upgrade | MuleSoft Mysore Meetup #46Mule 4.6 & Java 17 Upgrade | MuleSoft Mysore Meetup #46
Mule 4.6 & Java 17 Upgrade | MuleSoft Mysore Meetup #46
MysoreMuleSoftMeetup
 
The approach at University of Liverpool.pptx
The approach at University of Liverpool.pptxThe approach at University of Liverpool.pptx
The approach at University of Liverpool.pptx
Jisc
 
Model Attribute Check Company Auto Property
Model Attribute  Check Company Auto PropertyModel Attribute  Check Company Auto Property
Model Attribute Check Company Auto Property
Celine George
 
The French Revolution Class 9 Study Material pdf free download
The French Revolution Class 9 Study Material pdf free downloadThe French Revolution Class 9 Study Material pdf free download
The French Revolution Class 9 Study Material pdf free download
Vivekanand Anglo Vedic Academy
 
Welcome to TechSoup New Member Orientation and Q&A (May 2024).pdf
Welcome to TechSoup   New Member Orientation and Q&A (May 2024).pdfWelcome to TechSoup   New Member Orientation and Q&A (May 2024).pdf
Welcome to TechSoup New Member Orientation and Q&A (May 2024).pdf
TechSoup
 
Home assignment II on Spectroscopy 2024 Answers.pdf
Home assignment II on Spectroscopy 2024 Answers.pdfHome assignment II on Spectroscopy 2024 Answers.pdf
Home assignment II on Spectroscopy 2024 Answers.pdf
Tamralipta Mahavidyalaya
 

Recently uploaded (20)

Thesis Statement for students diagnonsed withADHD.ppt
Thesis Statement for students diagnonsed withADHD.pptThesis Statement for students diagnonsed withADHD.ppt
Thesis Statement for students diagnonsed withADHD.ppt
 
Digital Tools and AI for Teaching Learning and Research
Digital Tools and AI for Teaching Learning and ResearchDigital Tools and AI for Teaching Learning and Research
Digital Tools and AI for Teaching Learning and Research
 
S1-Introduction-Biopesticides in ICM.pptx
S1-Introduction-Biopesticides in ICM.pptxS1-Introduction-Biopesticides in ICM.pptx
S1-Introduction-Biopesticides in ICM.pptx
 
Azure Interview Questions and Answers PDF By ScholarHat
Azure Interview Questions and Answers PDF By ScholarHatAzure Interview Questions and Answers PDF By ScholarHat
Azure Interview Questions and Answers PDF By ScholarHat
 
The Challenger.pdf DNHS Official Publication
The Challenger.pdf DNHS Official PublicationThe Challenger.pdf DNHS Official Publication
The Challenger.pdf DNHS Official Publication
 
"Protectable subject matters, Protection in biotechnology, Protection of othe...
"Protectable subject matters, Protection in biotechnology, Protection of othe..."Protectable subject matters, Protection in biotechnology, Protection of othe...
"Protectable subject matters, Protection in biotechnology, Protection of othe...
 
BÀI TẬP BỔ TRỢ TIẾNG ANH GLOBAL SUCCESS LỚP 3 - CẢ NĂM (CÓ FILE NGHE VÀ ĐÁP Á...
BÀI TẬP BỔ TRỢ TIẾNG ANH GLOBAL SUCCESS LỚP 3 - CẢ NĂM (CÓ FILE NGHE VÀ ĐÁP Á...BÀI TẬP BỔ TRỢ TIẾNG ANH GLOBAL SUCCESS LỚP 3 - CẢ NĂM (CÓ FILE NGHE VÀ ĐÁP Á...
BÀI TẬP BỔ TRỢ TIẾNG ANH GLOBAL SUCCESS LỚP 3 - CẢ NĂM (CÓ FILE NGHE VÀ ĐÁP Á...
 
Group Presentation 2 Economics.Ariana Buscigliopptx
Group Presentation 2 Economics.Ariana BuscigliopptxGroup Presentation 2 Economics.Ariana Buscigliopptx
Group Presentation 2 Economics.Ariana Buscigliopptx
 
How to Make a Field invisible in Odoo 17
How to Make a Field invisible in Odoo 17How to Make a Field invisible in Odoo 17
How to Make a Field invisible in Odoo 17
 
Supporting (UKRI) OA monographs at Salford.pptx
Supporting (UKRI) OA monographs at Salford.pptxSupporting (UKRI) OA monographs at Salford.pptx
Supporting (UKRI) OA monographs at Salford.pptx
 
Unit 2- Research Aptitude (UGC NET Paper I).pdf
Unit 2- Research Aptitude (UGC NET Paper I).pdfUnit 2- Research Aptitude (UGC NET Paper I).pdf
Unit 2- Research Aptitude (UGC NET Paper I).pdf
 
Francesca Gottschalk - How can education support child empowerment.pptx
Francesca Gottschalk - How can education support child empowerment.pptxFrancesca Gottschalk - How can education support child empowerment.pptx
Francesca Gottschalk - How can education support child empowerment.pptx
 
TESDA TM1 REVIEWER FOR NATIONAL ASSESSMENT WRITTEN AND ORAL QUESTIONS WITH A...
TESDA TM1 REVIEWER  FOR NATIONAL ASSESSMENT WRITTEN AND ORAL QUESTIONS WITH A...TESDA TM1 REVIEWER  FOR NATIONAL ASSESSMENT WRITTEN AND ORAL QUESTIONS WITH A...
TESDA TM1 REVIEWER FOR NATIONAL ASSESSMENT WRITTEN AND ORAL QUESTIONS WITH A...
 
A Survey of Techniques for Maximizing LLM Performance.pptx
A Survey of Techniques for Maximizing LLM Performance.pptxA Survey of Techniques for Maximizing LLM Performance.pptx
A Survey of Techniques for Maximizing LLM Performance.pptx
 
Mule 4.6 & Java 17 Upgrade | MuleSoft Mysore Meetup #46
Mule 4.6 & Java 17 Upgrade | MuleSoft Mysore Meetup #46Mule 4.6 & Java 17 Upgrade | MuleSoft Mysore Meetup #46
Mule 4.6 & Java 17 Upgrade | MuleSoft Mysore Meetup #46
 
The approach at University of Liverpool.pptx
The approach at University of Liverpool.pptxThe approach at University of Liverpool.pptx
The approach at University of Liverpool.pptx
 
Model Attribute Check Company Auto Property
Model Attribute  Check Company Auto PropertyModel Attribute  Check Company Auto Property
Model Attribute Check Company Auto Property
 
The French Revolution Class 9 Study Material pdf free download
The French Revolution Class 9 Study Material pdf free downloadThe French Revolution Class 9 Study Material pdf free download
The French Revolution Class 9 Study Material pdf free download
 
Welcome to TechSoup New Member Orientation and Q&A (May 2024).pdf
Welcome to TechSoup   New Member Orientation and Q&A (May 2024).pdfWelcome to TechSoup   New Member Orientation and Q&A (May 2024).pdf
Welcome to TechSoup New Member Orientation and Q&A (May 2024).pdf
 
Home assignment II on Spectroscopy 2024 Answers.pdf
Home assignment II on Spectroscopy 2024 Answers.pdfHome assignment II on Spectroscopy 2024 Answers.pdf
Home assignment II on Spectroscopy 2024 Answers.pdf
 

1_5071503950121796163.pdf

  • 1. . 1 VISION IAS www.visionias.in TEST BOOKLET CSAT APTITUDE TEST– (4025) – 2023 Time Allowed: Two Hours Maximum Marks: 200 INSTRUCTIONS 1. IMMEDIATELY AFTER THE COMMENCEMENT OF THE EXAMINATION, YOU SHOULD CHECK THAT THIS BOOKLET DOES NOT HAVE ANY UNPRINTED OR TORN OR MISSING PAGES OR ITEMS ETC. IF SO, GET IT REPLACED BY A COMPLETE TEST BOOKLET. 2. ENCODE CLEARLY THE TEST BOOKLET SERIES A, B, C OR D AS THE CASE MAY BE IN THE APPROPRIATE PLACE IN THE ANSWER SHEET. 3. You have to enter your Roll Number on the Test Booklet in the Box provided alongside. DO NOT write anything else on the Test Booklet. 4. This Test Booklet contains 80 items (Questions). Each item is printed in English. Each item comprises four responses (answers). You will select the response which you want to mark on the Answer Sheet. In case you feel that there is more than one correct response, mark the response which you consider most appropriate. In any case, choose ONLY ONE response for each item. 5. You have to mark all your responses ONLY on the separate Answer Sheet provided. See direction in the answers sheet. 6. All items carry equal marks. Attempt all items. Your total marks will depend only on the number of correct responses marked by you in the answer sheet. For every incorrect response one-third of the allotted Marks will be deducted. 7. Before you proceed to mark in the Answer sheet the response to various items in the Test booklet, you have to fill in some particulars in the answer sheets as per the instruction sent to you with your Admission Certificate. 8. After you have completed filling in all responses on the answer sheet and the examination has concluded, you should hand over to Invigilator only the answer sheet. You are permitted to take away with you the Test Booklet. 9. Sheets for rough work are appended in the Test Booklet at the end. DO NOT OPEN THIS BOOKLET UNTIL YOU ARE ASKED TO DO SO C
  • 2. . 2 www.visionias.in ©Vision IAS 1. A tailor has to cut multiple rolls of fabric with a pair of scissors. Each roll of fabric has to be cut into 10 equal pieces. If he makes 45 cuts with his scissors per minute, how many rolls would be cut in 24 minutes? (a) 30 rolls (b) 58 rolls (c) 120 rolls (d) 150 rolls 2. A girl started counting the fingers of her left hand in the following manner: She started by calling the thumb 1, the index finger 2, middle finger 3, ring finger 4, little finger 5, and then reversed direction of counting calling the ring finger 6, middle finger 7 and so on. If she counted until 1994, she must have ended counting on which finger? (a) Ring finger (b) Middle finger (c) Thumb (d) Index finger 3. Choose a figure which would most closely resemble the unfolded form of Figure (Z). Select the answer using the correct code. (a) 1 (b) 2 (c) 3 (d) 4 4. Pinky was born on 29th Feb, 2016 which happened to be a Monday. On what day would she celebrate her birthday next? (a) Monday (b) Tuesday (c) Friday (d) Saturday 5. Find out which of the figures (1), (2), (3) and (4) can be formed from the pieces given in figure (X). (a) 1 (b) 2 (c) 3 (d) 4 6. What was the day of the week on 16th June, 1999? (a) Saturday (b) Monday (c) Wednesday (d) Thursday 7. A man has Rs. 480 in the denominations of one rupee notes, five rupee notes and ten rupee notes. The number of notes of each denomination is equal. What is the total number of notes that he has? (a) 45 (b) 75 (c) 90 (d) 120
  • 3. . 3 www.visionias.in ©Vision IAS Directions for the following 6 (six) items: Read the following five passages and answer the items that follow each passage. Your answers to these items should be based on the passages only. Passage – 1 New Zealand Cricket striking a deal to remunerate its women cricketers the same as their male counterparts is a major landmark in the fight to close the gender pay gap in sports. The United States women’s national footballers won six yearlong battles with their federation to secure equal compensation. The agreements are expected to be game changers, encouraging more girls to take up the sports. As New Zealand captain Sophie Devine said, “It is great to be recognized in the same agreement, alongside the men. It is a massive step forward and will be a huge drawcard for young women and girls.” But victories in equal pay struggles have not come easy. In cricket, any move to narrow the monetary gap between men and women, especially in India, is dithered over by citing lower market ratings for the ladies’ game. But it would be prudent to focus on the factors that are holding women back — unequal opportunities, curtailed playing time and lack of investment. Historically, men taking to sports and following sports have been organic exercises, largely because of social conditioning. 8. Which one of the following statements best reflects the critical message conveyed by the passage given above? (a) Achieving gender pay parity is an important step towards achieving gender equality, but challenges of social conditioning remain. (b) The need of the hour is to improve the social condition of women. (c) Reducing the pay gap is about being fair and respectful, and recognizing the effort and excellence women bring to the sport. (d) It is time the journey towards pay parity is hastened. 9. With reference to the above passage, the following assumptions have been made: 1. Society has nudged women to internalize that sporting participation is not for them, vis-à-vis men. 2. For women, it is trivial to be recognized at the same level as men. Which of the above assumptions are valid? (a) 1 only (b) 2 only (c) 1 and 2 (d) None Passage – 2 The capital city of the world’s largest democracy is driven by power, energy, and opportunities, dipped in nostalgia, rich in history and heritage and soaked in a myriad of compelling identities, memories, and emotions. The composite culture of thousands of years thrives even after the city was destroyed and rebuilt seven times. Each time it has risen like a phoenix from the ashes stamping the city with its genetic code distinctly seen in its architecture, monuments, museums, food, art, poetry, politics, culture, and language. The built environment of the city is a product of its changing socio-economic, cultural, and political forces. Those who live and work here lend to the city’s unique character, but its pollution levels, the game of one-upmanship, chaos and scare can be frustrating and infuriating. Yet, the city is counted among the most desirable ones to live in. Something is alluring and magical about it. 10. According to the passage, which among the following constructs a narrative of the city mentioned? 1. Symbolic imagery that addresses the dormant social, political and cultural climate. 2. Long eventful history of the city that saw the rise and fall of several empires. Which of the statements given above is/are correct? (a) 1 only (b) 2 only (c) 1 and 2 (d) None
  • 4. . 4 www.visionias.in ©Vision IAS Passage—3 In terms of things we keep, things that encapsulate particular moments of our past, it all depends on your relationship to memory, to how much you feel the need to live in the past, or how you use the past to fuel your present and future. I know people with vivid memories who have no need for the triggers or proof provided by objects. I also know people who have huge drop-outs in their memory but who won’t let go of a single dust- encrusted trinket. As for myself, all I know is that I have a newly deployed drawer for fresh hoarding and lots of magical elbowing and adjusting going on in other drawers as newly arrived things jostle with ones that have been occupying the space for years. 11. According to the passage, which of the following is correct about things we keep as souvenir? 1. They are necessary to remind people suffering from memory loss, about past events. 2. They keep lying idle in some drawers for years and serve no useful purpose. 3. They are most likely kept by those people who like to relive their pasts. Select the correct answer using the code given below. (a) 2 and 3 only (b) 3 only (c) 1 and 2 only (d) 1, 2 and 3 Passage – 4 The Governor of the Reserve Bank of India (RBI) has been reported as saying that the repo rate has been raised. Raising the interest rate in an attempt to control inflation, implicitly assumes that it reflects economy wide excess demand. To persist with monetary policy to curb inflation under these circumstances is to miss the point that, being a macroeconomic instrument, it cannot affect any particular price. raising interest rates is not going to solve the problem of inflation. It is not going to create more food. What you do is that you have supply-side interventions. Killing the economy through raising interest rates is not going to solve the inflation in any time frame. It is a common mistake to observe sharply rising prices of certain goods and conclude therefrom that it is this that is driving inflation. This conclusion can be way off the mark when the concerned goods account for only a small part of the consumption basket that the overall consumer price index is based on. 12. Which among the following is the most logical and rational inference that can be made from the passage given above? (a) Economists tend to pay adequate attention to supply side interventions. (b) Rather than correcting inflation by raising the interest rates, focus should be more on necessary supply-side interventions. (c) India’s economic agencies appear to have not fully understood what is driving inflation. (d) Monetary policy to curb inflation can never be effective. Passage – 5 Landslides are a common occurrence during the rainy season in hilly and mountainous regions. Heavy or continuous rain usually causes mud and rocks to come sliding down the mountain/hill, and in the process, destroy everything along their path. Houses, vehicles, people, trees, and animals often get buried in a landslide. When someone has a landslide victory in an election, he wins the contest by an overwhelming margin; he defeats his rivals soundly. What is it that a landslide does? It buries everything along its path. Someone who wins an election by a landslide destroys everything in his path; in this case, it suggests that the candidate does not just beat his opponents, he buries them! 13. Which one of the following statements best sums up the passage given above? (a) Sliding down the mountain/hill reflects winning over the opponent. (b) While it is true that a ‘landslide’ is usually associated with misfortune, a ‘landslide victory’ is seen as something evil. (c) While a ‘landslide’ is usually associated with something bad, during elections a big victory is called a landslide victory. (d) There is an elementary correlation between landslides in a mountainous region and big victories during the election.
  • 5. . 5 www.visionias.in ©Vision IAS 14. Six friends – A, B, C, D, E and F are sitting in two rows, three in each row. Each boy is sitting exactly opposite to another boy. A is to the North of D, and to the East of E. B is to the South-East of E and to South-West of A. F is not sitting to the West of B. Who is sitting in between C and D? (a) A (b) B (c) E (d) F 15. A clock is set right at 10 a.m. on Monday. It loses 15 minutes in 24 hrs. What will be the true time when the clock indicates 4 a.m. on the following Saturday? (a) 5:12 am (b) 5:32 am (c) 6:32 am (d) 5:48 am 16. The minute hand of a faulty clock overtakes the hour hand at intervals of 63 minutes of correct time. How many minutes in a day does the clock lose or gain? (a) 58(4/71) min (b) 54(6/81) min (c) 55(7/70) min (d) 56(8/77) min 17. In 2016, Mohan celebrated his birthday on Friday. Which will be the first year after 2016 when Mohan will celebrate his birthday on a Wednesday? (He was not born in January or February) (a) 2021 (b) 2023 (c) 2020 (d) 2025 Directions for following 2 (two) items: Consider the information given below and answer the items that follow. In a circus there are seven animals – Lion, Tiger, Deer, Rabbit, Cow, Giraffe and Crocodile. Lion, Tiger and Crocodile are carnivores; Deer, Rabbit, Cow and Giraffe are herbivores. For a play, a team of four animals has to be selected. Out of four, not more than two can be carnivores, and at least two must be herbivores. Further there are following restrictions to be kept in mind: 1. If Crocodile is included, then both Giraffe and Rabbit are excluded. 2. Lion and Tiger cannot be included together. 3. If Cow is included, then Deer cannot be included. 18. Which one of the following combinations is not possible? (a) Tiger, Giraffe, Deer and Rabbit (b) Lion, Cow, Giraffe and Rabbit (c) Crocodile, Cow, Rabbit and Tiger (d) Lion, Rabbit, Cow and Giraffe 19. If Tiger, Giraffe and Deer are selected, then which among the following can be the fourth member of the team? (a) Cow (b) Lion (c) Crocodile (d) Rabbit
  • 6. . 6 www.visionias.in ©Vision IAS Directions for the following 3 (three) items: Consider the information given below and answer the items that follow. Recently, Ghosh Babu spent his winter vacation on Kyakya Island. During the vacation, he visited the local casino where he came across a new card game. Two players, using a normal deck of 52 playing cards, play this game. One player is called the Dealer and the other is called the Player. First, the Player picks a card at random from the deck. This is called the base card. The amount in rupees equal to the face value of the base card is called the base amount. The face values of Ace, King, Queen and Jack are ten. For other cards, the face value is the number on the card. Once, the Player picks a card from the deck, the Dealer pays him the base amount. Then the Dealer picks a card from the deck and this card is called the top card. * If the top card is of the same suit as the base card, the Player pays twice the base amount to the Dealer. * If the top card is of the same colour as the base card (but not the same suit), then the Player pays the base amount to the Dealer. * If the top card happens to be of a different colour than the base card, the Dealer pays the base amount to the Player. Ghosh Babu played the game 4 times. * The first time he picked eight of clubs and the Dealer picked queen of clubs. * Second time, he picked ten of hearts and the dealer picked two of spades. * Next time, Ghosh Babu picked six of diamonds and the dealer picked ace of hearts. * Lastly, he picked eight of spades and the dealer picked jack of spades. Answer the following questions based on these four rounds. 20. What could have been the maximum gain made by Ghosh Babu, if he stopped playing at the right time (instead of playing all the four rounds)? (a) 12 (b) 20 (c) 16 (d) 4 21. The initial money that Ghosh Babu had was Rs. X (before the beginning of the game rounds). At no point did he have to borrow any money. What is the minimum possible value of X? (a) 16 (b) 8 (c) 100 (d) 24 22. If the final amount of money that Ghosh Babu had with him was Rs.100, what was the initial amount he had with him? (a) Rs. 120 (b) Rs. 8 (c) Rs. 4 (d) Rs. 96 Directions for the following 3 (three) items: Read the following three passages and answer the items that follow each passage. Your answers to these items should be based on the passages only. Passage—1 We are now in the middle of a long process of transition in the nature of the image which man has of himself and his environment. Primitive men, and to a large extent also men of the early civilizations, imagined themselves to be living on a virtually illimitable plane. That is, there was always someplace else to go when things got too difficult, either by reason of the deterioration of the natural environment or a deterioration of the social structure in places where people happened to live. Gradually, however, man has been accustoming himself to the notion of the spherical earth and a closed sphere of human activity where there is nowhere to go if things get difficult.
  • 7. . T h i s d o c u m e n t i s p e r s o n a l i s e d f o r A t i s h S h r e y a n s h ( a t i s h v i s i o n 2 0 0 2 @ g m a i l . c o m ) 7 www.visionias.in ©Vision IAS 23. With reference to the above passage, following assumptions have been made: 1. Only rise in population alters man’s image of himself and his environment. 2. Primitive men lacked the capability to accustom themselves to changed external situations – material or non-material. Which of the above assumptions is/are valid? (a) 1 only (b) 2 only (c) Both 1 and 2 (d) Neither 1 nor 2 Passage – 2 The discussion on the demerits of ‘freebies’ distributed to the public as a result of election promises is not new in India. However, there is often confusion on what constitutes ‘freebies,’ with several services that the Government provides to meet its constitutional obligations towards citizens also being clubbed in this category. The basic argument is that these are a waste of resources and place a burden on already stressed fiscal resources. In such discussions, ‘freebies’ not only include the free distribution of what may be considered ‘club goods’ such as televisions and gold chains but also welfare schemes such as free or subsidized rations under the Public Distribution System (PDS), cooked meals under the mid-day meal scheme, supplementary nutrition through anganwadis, and work provided through the Mahatma Gandhi National Rural Employment Guarantee Act (MGNREGA). MGNREGA for instance has been another scheme which has been a lifeline for many during the pandemic and earlier. At a time when there are few employment opportunities, working under MGNREGA can guarantee some assured wages; implemented in the true spirit of the legislation. 24. Concerning the above passage, the following assumptions have been made: 1. Making welfare delivery an electoral issue is a strategy adopted by politicians. 2. Freebies have both a positive and negative impact on the socio-economic growth of a nation. 3. Subsidy burden of the state also contributes to human development and protection of the basic rights of the people to nutrition, work and essentially the right to life with dignity. Which of the above assumptions is/are valid? (a) 1 only (b) 2 and 3 only (c) 3 only (d) 1, 2, and 3 Passage – 3 Violations of the right to education may occur through the direct action of States parties (the act of commission) or through their failure to take steps required by law (the act of omission). Whilst the vast majority of countries have ratified international treaties that recognize the full right to education, it is still denied to millions around the world due to a lack of resources, capacity, and political will. There are still countries that have not integrated the right to education into their national constitution or provided the legislative and administrative frameworks to ensure that the right to education is realized in practice. Most of the children and adults who do not fully enjoy the right to education belong to the most deprived and marginalised groups of society which are often left behind in national policies.
  • 8. . 8 www.visionias.in ©Vision IAS 25. Based on the above passage, the following assumptions have been made: 1. Integration of the right to education in national laws will end the violations of the right to education. 2. Violations of the right to education can be done by state parties. Which of the above assumptions is/are valid? (a) 1 only (b) 2 only (c) 1 and 2 only (d) None 26. Tuesday fell on which of the following dates of April, 2002? (a) 3rd, 10th, 17th, 24th (b) 1st, 8th, 15th, 22nd, 29th (c) 4th, 11th, 18th, 25th (d) 2nd, 9th, 16th, 23rd, 30th 27. Consider the following figure. Find the minimum number of straight lines required to make the given figure. (a) 28 (b) 26 (c) 23 (d) 24 28. Sheetal went to a ‘Sale’ to purchase some pants and skirts. When her friends asked her how many of each she had bought, she answered, “The number of skirts is two less than twice the number of pants purchased. Also, the number of skirts is four less than four times the number of pants purchased”. What is the number of pant and skirts purchased by Sheetal? (a) 2, 2 (b) 1, 0 (c) 1, 1 (d) 4, 2 29. Consider the following pattern of numbers: 16 18 22 26 8 9 10 13 20 16 10 11 28 25 22 ? What is the missing number in the above pattern? (a) 19 (b) 24 (c) 26 (d) 28 30. The time in a clock is 20 minute past 2. Find the angle between the hands of the clock. (a) 60 degrees (b) 120 degrees (c) 45 degrees (d) 50 degrees 31. How often between 11 O'clock and 12 O'clock are the hands of the clock together? (a) 5 (b) 2 (c) 3 (d) 0
  • 9. . 9 www.visionias.in ©Vision IAS 32. 27th February, 2003 was a Thursday. What must have been the day on 27th February, 1603? (a) Monday (b) Thursday (c) Sunday (d) Tuesday 33. A clock loses 1% of the week-time during the first week and then gains 2% of the week-time during the next one week. If the clock was set right at 12 noon on a Sunday, what will be the time that the clock will show exactly 14 days from the time it was set right? (a) 1: 36 (b) 1: 40 (c) 1: 41 (d) 1: 19 34. Select the option in which the given figure is embedded. (Rotation or Flipping is not allowed). (a) (b) (c) (d) Directions for the following 4 (four) items: Consider the information given below and answer the items that follow. Cinemax is a very popular multiplex in the city of Mumbai. It has five screens – Screen 1, Screen 2, Screen 3, Screen 4 and Screen 5. Five different movies – Predestination (PN), Looper (LR), Interstellar (IR), Inception (IN) and Equalizer (ER) – were screened over five consecutive days on these screens. Each movie was shown on a different screen every day, i.e. every movie was shown on each screen exactly once. Similarly, each screen showed a different movie every day, i.e. every screen showed a particular movie exactly once. Further, the following information is known: i. In all the screens, except screen 2, IR was screened just before IN. ii. LR was the third movie to be screened on screen 4, and it was screened two days after PN. iii. PN was screened on two other screens prior to screen 3, and was screened on 1 immediately after it was screened on 3. iv. IN was released on screen 3 on day 2. v. On day 1, PN and ER were released on two consecutive screen numbers. 35. Which movie was screened on the first day on screen 2? (a) ER (b) IN (c) PN (d) Cannot be determined 36. On which screen was IR screened on the fourth day? (a) Screen 4 (b) Screen 1 (c) Screen 2 (d) Cannot be determined
  • 10. . 10 www.visionias.in ©Vision IAS 37. Which of the following is the correct order of screens (from first day to fifth day) on which Equalizer was screened? (a) 4, 1, 2, 3, 5 (b) 3, 5, 1, 2, 4 (c) 5, 4, 2, 3, 1 (d) Cannot be determined 38. On which screen was IN screened immediately on the next day after it was screened on screen 1? (a) 4 (b) 5 (c) 2 (d) Cannot be determined Directions for the following 3 (three) items: Read the following two passages and answer the items that follow each passage. Your answers to these items should be based on the passages only. Passage – 1 Climate change and its impacts across the globe will threaten the bottom line of businesses in a variety of ways. The frequency and intensity of extreme weather in countries, can damage factories, supply chain operations and other infrastructure, and disrupt transport. Drought will make water more expensive, which will likely affect the cost of raw materials and production. Climate volatility may force companies to deal with uncertainty in the price of resources for production, energy transport and insurance. And some products could become obsolete or lose their markets, such as equipment related to coal mining or skiing in an area that no longer has snow. Whether in the U.S. or abroad, new regulations such as carbon pricing and subsidies that favour a competitor may affect a business’s bottom line. A company’s reputation could also suffer if it’s seen as doing something that hurts the environment. And investors and stakeholders are increasingly worried about the potential for “stranded assets”—those that become prematurely obsolete or fall out of favour and must be recorded as a loss, such as fossil fuels that many believe should stay in the ground or real estate in a newly designated flood plain. 39. Which among the following are the possible disadvantages of climate change? 1. Increased prices of goods. 2. Forcing companies to think about the environment. 3. Redundancy of products. 4. Introduction of carbon pricing. Select the correct answer from the code given below: (a) 1 and 3 only (b) 3 and 4 only (c) 2 and 4 only (d) 1 and 2 only 40. Which one of the following statements best reflects the crux of the passage? (a) Climate change will drive the change in the pattern of future production of resources. (b) Companies which no longer focus on climate governance will face a boycott of their goods. (c) Nations across the globe need to tackle climate change by making it their number one priority. (d) Only focusing on sustainable development can help us cope with the menace of climate change. Passage—2 The root of sufferings is neither the feeling of pain nor of sadness nor even of meaninglessness. Rather, the real root of suffering is this never-ending and pointless pursuit of ephemeral feelings, which causes us to be in a constant state of tension, restlessness and dissatisfaction. Due to this pursuit, the mind is never satisfied. Even when experiencing pleasure, it is not content, because it fears this feeling might soon disappear, and craves that this feeling should stay and intensify. People are liberated from sufferings not when they experience this or that fleeting pleasure, but rather when they understand the impermanent nature of all their feelings, and stop craving for them.
  • 11. . 11 www.visionias.in ©Vision IAS 41. What is the most logical and rational inference that can be made from the above passage? (a) As feelings are never-ending and ephemeral, sufferings can never end and human mind cannot ever be satisfied. (b) Feelings of pain, sadness and meaninglessness, not in isolation, but in combinations of two or three are at the root of sufferings. (c) Gautam Buddha’s observations on sufferings in this world are universal and timeless. (d) None of the above 42. What is the angle between the minute hand and the hour hand when the time is 15:40 hours? (a) 150 (b) 160 (c) 140 (d) 130 43. The first two problem figures given below follow a certain pattern. The same pattern should be followed by the next two problem figures. Choose the correct answer figure that can replace the question mark. Problem figures Answer figures (a) 1 (b) 2 (c) 3 (d) 4 44. Out of the four figures marked (a) (b) (c) and (d), three are similar in a certain manner. However, one figure is not like the other three. Choose the figure which is different from the rest. (a) a (b) b (c) c (d) d Directions for the following 4 (four) items: Consider the information given below and answer the items that follow. Kumar has a peach orchard in Ooty. On a particular day, his associate Suresh plucked 103 distinct peaches (each peach of a distinct size). He placed these 103 peaches in four distinct boxes – A, B, C and D at 6 AM. Later at multiple times during the day (8 AM, 10 AM, 12 noon and 2 PM), Kumar tested some of the peaches in some or all of the four boxes. He performed one of the following three operations – I, II and III on the peaches he tested: Operation I: If Kumar liked any peach in a box, he plucked a new peach of the same size and placed it in one of the other three boxes. Operation II: If Kumar did not like the box in which a particular peach was placed, he removed the peach from that box and placed it in another box. Operation III: If Kumar did not like a particular peach, he removed the peach from the box and threw it away. The following table provides information about the number of peaches in each of the boxes – A, B, C and D at different times during the entire day. Kumar was very lazy and hence performed minimum possible number of operations. An operation is said to have been performed, when any of I, II or III is performed. Boxes 6 AM 8 AM 10 AM 12 Noon 2 PM A 23 25 29 26 28 B 27 31 31 32 29 C 15 27 40 39 32 D 38 30 25 24 45
  • 12. . 12 www.visionias.in ©Vision IAS 45. Find the total number of operations performed by Kumar between 6:00 AM and 2:00 PM. (a) 62 (b) 67 (c) 63 (d) 65 46. How many times did Kumar perform operation III between 6:00 AM and 2:00 PM? (a) 6 (b) 4 (c) 5 (d) 3 47. At least how many times was operation II performed to remove a peach from box D and place it in box C between 6:00 AM and 2:00 PM? (a) 1 (b) 2 (c) 3 (d) More than 3 48. How many times did Kumar perform operation II between 6:00 AM to 2:00 PM? (a) 24 (b) 25 (c) 23 (d) 22 Directions for the following 4 (four) items: Read the following two passages and answer the items that follow each passage. Your answers to these items should be based on the passages only. Passage – 1 With the rise of packaged food, food with high salt and sugar content, obesity-related malnutrition is on the rise. India is now facing a double whammy of malnutrition, with the younger population being affected by undernutrition and the older population being affected by malnutrition related to obesity. We currently have no significant programmes that target malnutrition affected by obesity. As a country, we could take the community nutrition approach where nutrition is improved at the community level. One of the most successful examples of this approach in the country is the addition of iodine to salt to prevent goitre and intellectual disabilities. This approach includes educating people about dietary changes and helping them achieve optimum levels of nutrition. An important part of this is educating children at schools and people in shared spaces such as workplaces. The main aim of this approach is preventive. 49. Based on the above passage, the following assumptions have been made: 1. It is not possible to overcome malnutrition by obesity as community nutrition is preventive in nature. 2. Absence of government programmes for fighting malnutrition is the primary cause of its existence. 3. People should be encouraged to eat fresh and nutrition rich food instead of packaged or high-salt foods. Which of the above assumptions is/are valid? (a) 1 and 3 only (b) 3 only (c) 1, 2 and 3 (d) 1 and 2 only
  • 13. . 13 www.visionias.in ©Vision IAS 50. Which one of the following statements best reflects the message of the author of the passage? (a) With the success of fighting goitre, the community nutrition approach is the best approach for fighting malnutrition by obesity. (b) Considering the ill consequences, India should focus on handling malnutrition by obesity using the community nutrition approach. (c) Government needs to devise more effective programmes for fighting malnutrition by obesity. (d) Companies manufacturing packaged food, food with high salt and sugar content should be adequately regulated. Passage – 2 Invasive species threaten biodiversity globally, and invasive mammalian predators are particularly damaging, having contributed to considerable species decline and extinction. Invasive predators are implicated in 87 birds, 45 mammals, and 10 reptile species extinctions - 58% of these groups’ contemporary extinctions worldwide. These figures are likely underestimated because 23 critically endangered species that were assessed are classed as “possibly extinct.” Invasive mammalian predators endanger a further 596 species at risk of extinction, with cats, rodents, dogs, and pigs threatening the most species overall. Species most at risk from predators have high evolutionary distinctiveness and inhabit insular environments. Invasive mammalian predators are therefore important drivers of the irreversible loss of phylogenetic diversity worldwide. That most impacted species are insular indicates that management of invasive predators on islands should be a global conservation priority. 51. Which of the following is/are the most rational and logical Inference/Inferences that can be made from the passage? 1. Invasive species along with being dangerous for the environment also, have some potential benefits for it. 2. Island species are the most vulnerable to extinction due to invasive predators. Select the correct answer from the code given below: (a) 1 only (b) 2 only (c) Both 1 and 2 (d) Neither 1 nor 2 52. Which one of the following statements best reflects the crux of the passage? (a) To ensure the survival of the human race, the conservation of biodiversity is the most important task. (b) Countries across the world should fund biodiversity conservation efforts on Islands. (c) Invasive species can grow anywhere and are always endangering the endemic species. (d) Invasive predators have the potential to change the species diversity of an ecosystem. 53. In a caravan, in addition to 50 hens, there are 45 goats, 8 camels and some human keepers. If the total number of feet are 224 more than the number of heads, the number of keepers must be (a) 5 (b) 8 (c) 10 (d) 15
  • 14. . 14 www.visionias.in ©Vision IAS 54. Aruna cuts a cake into two halves and then further cuts one of the halves into smaller pieces of equal size. Each of the smaller pieces is twenty grams in weight. If she has seven pieces of the cake in all with her, what must have been the total weight of the original cake? (a) 100 grams (b) 150 grams (c) 200 grams (d) 240 grams 55. At what time between 1 and 2 o’ clock will the hands of a watch make an angle of 180°? (a) 35(5/11) min. past 1 (b) 40 min. past 1 (c) 50(4/11) min. past 1 (d) 38(2/11) min. past 1 56. Which of the answer figures can replace the question mark in the problem figure? (a) D (b) C (c) A (d) B 57. Out of the given answer figures, which is the correct one to replace the empty box? (a) a (b) b (c) c (d) d 58. If 28th August, 1946 was a Wednesday; what day of the week must have fallen on 31st August, 1961? (a) Tuesday (b) Thursday (c) Monday (d) Wednesday 59. If Palmyra < Date plam; Date plam < Queen palm and Queen palm > Caryota, then which of the following is always true? (a) Palmyra > Caryota (b) Palmyra < Queen palm (c) Date palm = Caryota (d) Date palm > Caryota
  • 15. . 15 www.visionias.in ©Vision IAS Direction for the following 3 (three) items: Consider the information given below and answer the items that follow. In a school, there are exactly six classrooms in a straight line. In these classrooms six different classes are held, namely for class I, II, III, IV, V and VI. In the first period exactly six different subject teachers - Hindi, English, Science, Math, Drawing and Sports - went to these classes, such that only one teacher went to each class. The following information is also known about the teachers who went to the various classes: 1. Drawing teacher went to the class to the immediate left of class V and to the immediate right of class II. 2. Math teacher went to class III which is adjacent to class II. 3. English teacher went to the class to the immediate left of class I. 4. Hindi teacher went to class IV, which is first from the left end and to the immediate left of the class to which Science teacher went. 5. There are exactly four classes to the left of class l and class IV is not adjacent to class III. 60. Math teacher went to which class? (a) V (b) VI (c) III (d) II 61. Which subject teacher went to class VI? (a) Science (b) Hindi (c) Drawing (d) Sports 62. Which subject’s teacher went to the class, which is to the immediate right of class II? (a) Science (b) Hindi (c) Drawing (d) Math Directions for the following 4 (four) items: Read the following three passages and answer the items that follow each passage. Your answers to these items should be based on the passages only. Passage – 1 The food system and agriculture are deeply interrelated with land, water, and energy. Agriculture in the U.S. is an energy-intensive sector and a major user of ground and surface water. Direct energy use in agriculture is primarily from fossil fuels used to operate automobiles and machinery for preparing fields, planting, and harvesting crops, applying chemicals, and transporting inputs and outputs to and from the market. Electricity is used largely for lighting, heating, cooling, and operation of in-farm equipment. Indirect energy is consumed off the farm for manufacturing fertilizers and pesticides, with the production of fertilizer being “extremely energy-intensive, requiring large amounts of natural gas”. Currently, there are U.S. public programs that promote renewable energy options and recommendations in conserving fuel and energy for farms, efficient use of fertilizers and pesticides, soil conservation measures, and energy-efficient methods of growing and transporting foods, but those programs will need high levels of participation to curb both climate impacts as well as address air and water quality issues associated with traditional agricultural practices. 63. Based on the above passage, the following assumptions have been made: 1. Future of agriculture in the U.S. is bleak due to dependence on fossil fuels. 2. Only the replacement of fossil fuels with renewable energy may not solve the problem of air and water pollution caused by agriculture. Which of the above assumptions is/are valid? (a) 1 only (b) 2 only (c) Both 1 and 2 (d) Neither 1 nor 2
  • 16. . 16 www.visionias.in ©Vision IAS 64. Which among the following are the possible uses of energy in agriculture? 1. Preparation of fields for growing crops. 2. Transportation of inputs to and outputs from the field. 3. Application of fertilizers. Select the correct answer from the code given below: (a) 1, 2 and 3 (b) 1 and 2 only (c) 2 and 3 only (d) 1 and 3 only Passage - 2 A National Green Tribunal (NGT) bench headed by Justice Sudhir Agarwal on September 28, 2022 ordered mining operations at the Shaliganga Nallah in the Budgam district of the Kashmir Valley to halt, calling into question the environmental clearances (EC) given for the operations. The order stated that according to the appellant, “All the three proposals were rejected by the (administration-appointed) Jammu and Kashmir Expert Appraisal Committee (JKEAC) on the ground that the area for which ECs were applied, are already overexploited; depleted to a large extent due to heavy illegal mining.” “Shaliganga Nallah is a feeding channel for the Hokersar wetland and mining activity may be detrimental to natural flow and quality of water flowing into Hokersar; irrigation Kuhls are also present within the mining site,” it added. Kuhls are traditional, community-managed irrigation systems in Kashmir and Himachal Pradesh. The JKEAC examined the mining blocks on the latest Google Image of October 2021. It found that minerals had been depleted to a large extent due to heavy illegal mining. Hardly any material was available for exploitation without endangering the environmental setting of the Nallah. 65. Which of the following is/are the most rational and logical Inference/ Inferences that can be made from the passage? 1. Technology is assisting the government with environmental protection. 2. The Himalayas' traditional and locally run irrigation system is endangered by mining. 3. Water contamination and impairment of water flow may be caused by mining activities. Select the correct answer using the codes given below. (a) 2 only (b) 2 and 3 only (c) 3 only (d) 1, 2 and 3 Passage – 3 In just the last three centuries, global forest areas have shrunk by 40 per cent. Every year, to meet the timber needs from natural sources, the Earth is stripped of 100 million trees. They store 50 per cent of the world’s terrestrial carbon and provide a buffer from extreme weather, such as hurricanes and tsunamis. The “State of the World’s Trees” assessment evaluated 60,000 tree species and found that 30 per cent are at the risk of extinction. Extinction in the plant kingdom is “twice the number of threatened tree species globally than threatened mammals, birds, amphibians and reptiles combined”. Over 440 tree species are on the brink of extinction, meaning they have fewer than 50 individuals remaining in the wild, the report reveals. The rapidly snowballing loss in diversity is largely triggered and fuelled by new age emergencies and interventions, like climate change, change in atmospheric composition and the spread of invasive alien species. The current warming is 1.2°C above the pre-industrial levels. This means, from the evolutionary perspective, humans are already moving out of the Holocene environment that ensured the right temperature for us to evolve and take up farming. The IPCC report cites that half of all species are moving towards the poles or to a higher elevation to adapt to the new planetary climate. At the sea, due to the warming, species have travelled poleward at the rate of 59 km per decade on average.
  • 17. . 17 www.visionias.in ©Vision IAS 66. Biodiversity loss is due to which of the following factors? 1. Spread of alien invasive species 2. Global warming and climate change 3. Deforestation Which of the above assumptions is/are valid? (a) 1 and 2 only (b) 2 and 3 only (c) 1 and 3 only (d) 1, 2 and 3 67. A bus starts from city X. The number of women in the bus is half of the number of men. In city Y, 10 men leave the bus and five women enter. Now the number of men and women becomes equal. In the beginning, how many passengers entered the bus in city X? (a) 25 (b) 35 (c) 45 (d) 55 68. Consider the following equation: 5# × 3#0 ÷ #0 – 3#3 + #3 = 3# In the above equation what does the symbol # stands for? (a) 4 (b) 5 (c) 8 (d) 6 69. What is the number of days in x weeks and x days? (a) 7x + 1 (b) 8x (c) 15x (d) 7x + 7 70. If yesterday were tomorrow, then today would be Friday. What day is today? (a) Saturday (b) Monday (c) Sunday (d) Tuesday Directions for the following 4 (four) items: Consider the information given below and answer the items that follow. O, P, Q, R, S, T, U, V, W, X, Y and Z are 12 persons, 3 each from four different families – the Ahuja’s, the Sharma’s, the Malhotra’s and the Mishra’s. Given below are five groups, each group consisting of four persons, such that no group consists of two persons from the same family. Group A O Q R S Group B Q W Y Z Group C R T V X Group D O U V Z Group E P U X Y Further it is given that: I. R and Y are not from the same family. II. T and Y are not from the same family. III. O is from the Ahuja’s. IV. Neither T nor Y are from the Sharma’s. V. T is not from the Ahuja’s. 71. X is from the same family as: (a) Q (b) W (c) Z (d) Cannot be determined
  • 18. . 18 www.visionias.in ©Vision IAS 72. Who among the following is definitely from the Sharma’s? (a) Z (b) P (c) Both Z and P (d) either Z or P 73. Which of the following statements is not correct? (a) If Q is from the Mishra’s, then S is from the Malhotra’s. (b) X is from the Ahuja’s and Z is from the Sharma’s. (c) If U is from the Malhotra’s, then S is from the Mishra’s. (d) None of these. 74. For how many persons, the exact families they belong to can be uniquely determined? (a) 10 (b) 9 (c) 8 (d) 6 75. When Rahul was born, his father was 32 years older than his brother and his mother was 25 years older than his sister. If Rahul’s brother is 6 years elder to him and his mother is 3 years younger than his father, then how old was Rahul’s sister when he was born? (a) 5 years (b) 8 years (c) 10 years (d) 14 years Directions for the following 5 (five) items: Read the following four passages and answer the items that follow each passage. Your answers to these items should be based on the passages only. Passage – 1 The societal pressure to pursue happiness ironically seems to have adverse effects on people's psychological well-being. Especially in countries that score high on the World Happiness Index (WHI), there is a strong link between the feeling of needing to be happy and the extent to which people actually experience feelings such as sadness, gloom, fatigue or anxiety. 'The pursuit of happiness' prevails everywhere, on social media, in self-help books and in advertisements. Because you only see seemingly happy people, it is extra conspicuous if you deviate from the happiness norm. As a result, people who don't meet the standard can have a more negative self-image and feel even worse. Negative emotions often seem unacceptable. There is still a stigma on sadness, grief, and fear. 76. On the basis of the passage given above, the following assumptions have been made: 1. In countries that score low on the World Happiness Index (WHI), there is a weak link between the feeling of needing to be happy and the extent to which negative emotions are experienced. 2. Achieving societal norms of happiness will lead to happiness in life. Which of the above assumptions is/are valid? (a) 1 only (b) 2 only (c) 1 and 2 both (d) Neither 1 nor 2
  • 19. . 19 www.visionias.in ©Vision IAS Passage – 2 The principle that patients should be autonomous is a central tenet of medical law and ethics. Autonomy gives rise to certain interests and certain rights: the interests of self-determination and bodily integrity, and the moral and legal right to consent to and to refuse medical treatment. Naturally, the autonomous cosmetic surgery patient shares this interest in self-determination and bodily integrity and thus, the moral and legal right to consent to and to refuse treatment. The patient should be able to refuse treatment, but how far should the doctor be able to refuse to give treatment asked for? Ideas of autonomy have not only been related to the individual self-expression of the patient; indeed, they have been used to validate arguments for clinical independence and discretion. O’Neill has argued, following Kant, that autonomy goes beyond individualism and self-expression and involves the exercise of duties, obligations, and respect for other moral agents and their own rights. 77. Which of the following statements best reflects the message of the author of the passage? (a) Every patient has the right to get treated by the doctor. (b) Patients are moral agents that exercise their rights over their body, and such autonomy can never be violated. (c) Medical law and ethics attempts to reconcile the dichotomy between patient's rights and doctor's rights and clinical independence. (d) In medical law and ethics, bodily integrity, self-determination, self-expression and individualism are cardinal. Passage - 3 The United Arab Emirates’s (UAE)’s Advanced Visa System, first announced by the Federal Authority for Identity, Citizenship, Customs and Ports Security (ICP) last month, will come into effect on Monday (October 3). Approved by UAE’s Cabinet in April, the new visa rules — aimed at reforming the country’s immigration and residency policies — include changes such as longer visas for tourists, extended residency for professionals under the Green Visa and an expanded 10-year Golden Visa scheme. Besides alleviating burdens for foreigners and simplifying visa procedures, the updated visa system is an attempt at “improving the quality of life and making the experience of living, working and investing in the UAE a pleasant and happy one”, said Major General Sultan Yousef Al Nuaimi, the Director General of Residency and Foreigners Affairs, as reported by Gulf News. Tourist visas will now allow visitors to legally enter and stay in the UAE for 60 days, an increase from the previous 30 days. A five- year, flexible multi-entry tourist visa was also introduced that allows them to stay in the UAE for up to 90 days in a row. Furthermore, the job exploration visa, which seeks to easily allow talented professionals to find employment in the UAE, will not require a sponsor or host. Those that fall under the first, second or third skill level classification of the Ministry of Human Resources and Emiratisation and new graduates from the top 500 universities of the world will be eligible for the job exploration visa.
  • 20. . 20 www.visionias.in ©Vision IAS 78. Which of the following is/are the most rational and logical Inference/ Inferences that can be made from the passage? 1. The new visa system was put into place in order to promote increased investment in the UAE. 2. Foreigners can stay in the UAE for up to 4 months straight, thanks to the introduction of a multi-entry tourist visa. Select the correct answer using the codes given below. (a) 1 only (b) 2 only (c) Both 1 and 2 (d) Neither 1 nor 2 Passage – 4 Since there are multiple truths and multiple perspectives in sociology, it is hardly surprising that there are also multiple methods. There is no single unique road to sociological truth. Of course, different methods are more or less suited to tackle different types of research questions. Moreover, every method has its own strengths and weaknesses. It is thus futile to argue about the superiority or inferiority of different methods. It is more important to ask if the method chosen is the appropriate one for answering the question that is being asked. For example, if one is interested in finding out whether most Indian families are still ‘joint families’, then a census or survey is the best method. However, if one wishes to compare the status of women in joint and nuclear families, then interviews, case studies or participant observation may all be appropriate methods. There are different ways of classifying or categorising various methods commonly used by sociologists. It is conventional, for example, to distinguish between quantitative and qualitative methods: the former deals in countable or measurable variables (proportions, averages, and the like) while the latter deals with more abstract and hard to measure phenomena like attitudes, emotions and so on. A related distinction is between methods that study observable behaviour and those that study non-observable meanings, values and other interpretational things. 79. Which of the following is/are the most rational and logical Inference/ Inferences that can be made from the passage? 1. Participant observation is the best method to use if one wants to determine whether the majority of Indian families are still considered "joint families." 2. There is no single, objective method in sociology because there are many competing truths and perspectives. Select the correct answer using the codes given below. (a) 1 only (b) 2 only (c) Both 1 and 2 (d) Neither 1 nor 2 80. Which one of the following is the most important implication to the above passage? (a) It's crucial to select the one right research approach to get accurate sociological findings. (b) The path to sociological truth is not one- size-fits-all. (c) The status of women has improved in nuclear families. (d) For the most portion, qualitative research methods are used in sociological procedures.
  • 21. . 1 www.visionias.in ©Vision IAS VISIONIAS www.visionias.in ANSWERS & EXPLANATION APTITUDE TEST–Test (4025) – 2023 1.(c) Number of cuts needed to cut a roll into 10 pieces = 9. Therefore, Number of rolls cut = 24 × 45/9 = 120. 2.(d) Clearly, while counting, the numbers associated with the thumb are 1, 9, 17, 25,………, i.e., numbers of the form (8n + 1). Since 1994 = 249 × 8 + 2, so 1993 shall correspond to the thumb and 1994 to the index finger. 3.(a) 4.(d) 29th Feb, 2016 = Monday. So, 28th Feb, 2016 = Sunday 28th Feb, 2017 = Tuesday (because 2016 is a leap year, there will be 2 odd days) Therefore, Feb 28th 2018 (Wednesday), Feb 28th 2019 (Thursday), Feb 28th 2020 (Friday), Feb 29th 2020 (Saturday) 5.(c) 6.(c) 16th June, 1999 = (1998 years + time period from 1.1.1999 to 16.6.1998) 1600 years » 0 odd days 300 years » 1 odd day 98 years has 74 ordinary years + 24 leap years. Number of odd days in 98 years (24 × 2 + 74) = 122 days = 3 odd days. Now, January + February + March + April + May + June of 1999 = (31 + 28 + 31 + 30 + 31 + 15) = 166 days = 23 weeks & 5 days = 5 odd days. So, Total number of odd days = (0 + 1 + 3 + 5) = 2 So, the given day is Monday + 2 = Wednesday. 7.(c) Let the number of notes of each denomination be x. Then, x + 5x + 10x = 480 Or 16x = 480 Or x = 30 Hence, total number of notes = 3x = 90 8.(a) Option (a) is correct. The passage mentions, “striking a deal to remunerate its women cricketers the same as their male counterparts is a major landmark in the fight to close the gender pay gap in sports.,” “But it would be prudent to focus on the factors that are holding women back — unequal opportunities,
  • 22. . 2 www.visionias.in ©Vision IAS curtailed playing time and lack of investment. Historically, men taking to sport and following sport have been organic exercises, largely because of social conditioning.” It implies achieving gender pay parity is an important step towards gender equality. But the social conditioning is still holding them back. So, this option is correct. Option (b) is incorrect. The statement only talks about improving the social condition of women. It does not bring about the issue of the gender pay gap. So, this option does not best capture the essence of the passage in its entirety. Therefore, this option is incorrect. Option (c) is incorrect. The statement only recognizes the change brought about by a reduced gender pay gap. It does not incorporate social conditioning facets of women. So, this statement is incorrect. Option (d) is incorrect. This statement only talks about pay parity. The passage is more holistic in nature when it comes to gender-gap issue. 9.(a) Assumption (1) is correct. The passage mentions, “In cricket, any move to narrow the monetary gap between men and women, especially in India, is dithered over by citing lower market ratings for the ladies’ game. But it would be prudent to focus on the factors that are holding women back — unequal opportunities, curtailed playing time and lack of investment. Historically, men taking to sport and following sport have been organic exercises, largely because of social conditioning.” It implies, through social conditioning, women have been strained to internalize that sporting participation is not that significant an activity for them. So, this assumption is correct. Assumption (2) is incorrect. The passage mentions, “It’s great to be recognized in the same agreement, alongside the men.” It highlights the importance of placing men and women at the same pedestal. As “trivial” signifies something of little value, this assumption is incorrect. 10.(b) Statement (1) is incorrect. The passage mentions, “The built environment of the city is a product of its changing socio­economic, cultural, and political forces.” Here, the author mentioned “changing”, not “dormant” or “stagnant”, social, political, and cultural climate. So, this option is incorrect. Statement (2) is correct. The passage mentions, “The composite culture of thousands of years thrives even after the city was destroyed and rebuilt seven times. Each time it has risen like a phoenix from the ashes stamping the city”. It implies a long eventful history of several rises and falls. This statement builds the narrative of the city. So, this option is correct. 11.(b) Souvenir: a thing that is kept as a reminder of a person, place, or event. Statement 1 is incorrect. The passage does not limit its scope to a section of people suffering from memory loss. Statement 2 is incorrect as the passage nowhere says that these souvenirs serve no purpose. The passage in the last sentence just says that they “have been OCCUPYING THE SPACE for years”. Also, the tone of this passage is not against the practice of keeping such things. Author quotes potential advantages of such souvenirs like their role in helping us use the past to fuel our present and future. Statement 3 is correct. The passage in the 1st sentence says that “it all depends on your relationship to memory…” and “ to how much you feel the need to live in the past”. Hence, it is likely that people desirous of reliving their pasts keep such things. 12.(b) Option (a) is incorrect. In the passage, the author is suggesting that more attention is paid to managing interest rates via the monetary policy, expressing the need to instead focus upon supply-side interventions in the macroeconomic policy. So, this option is incorrect. Option (b) is correct. The passage’s main purpose is to bring to light how the misconception that raising the interest rates would cure inflation is leading to a lack of attention to necessary supply-side interventions. So, this is the most logical and rational inference that can be made from the passage. So, this option is correct.
  • 23. . 3 www.visionias.in ©Vision IAS Option (c) is incorrect. This option is partially incorrect, because the passage nowhere questions the understanding of the economic agencies. It only questions the focus and steps being taken, suggesting a re-direction of efforts. So, this option is incorrect. Option (d) is incorrect. This option is not correct. Monetary policy might not be effective if it leaves out supply-side interventions. But we cannot infer that monetary policy can never be effective in curbing inflation. This passage is suggesting that “To persist with monetary policy to curb inflation under these circumstances is to miss the point”. So, this option is incorrect. 13.(c) Option (a) is incorrect. The statement is absurd. So, this option is incorrect. Option (b) is incorrect. The passage mentions, “in the process, destroy everything along their path. Houses, vehicles, people, trees, and animals often get buried in a landslide.,” “When someone has a landslide victory in an election, he wins the contest by an overwhelming margin; he defeats his rivals soundly.” It implies, landslides bring unfortunate circumstances along with them, destroy everything in their path etc. The author compared a landslide to an election victory with an overwhelming margin, which leaves opponents quite far back. This kind of victory is good, and it should be celebrated. So, this option is incorrect. Option (c) is correct. The passage mentions, “Heavy or continuous rain usually causes mud and rocks to come sliding down the mountain/hill, and in the process, destroy everything along their path,” and “When someone has a landslide victory in an election, he wins the contest by an overwhelming margin; he defeats his rivals soundly.” The author compared a landslide to a big victory in the passage. While a landslide is a misfortune, a landslide victory is a celebration. So, this option is correct. Option (d) is incorrect. The passage does not suggest that there is any correlation between landslides in a mountainous region and big victories during the election. 14.(b) A is to the North of D and to the East of E. B is to the South–East of E and South–West of A. The above data can be represented as follows: F is not sitting to the West of B. Hence, F must be to the North of B, and C to the West of B. Hence, the final arrangement will be: Therefore, B is sitting in between C and D. 15.(a) From 10 am on 1st day to 4 am on 5th day there are a total of 114 hours. As the clock loses 15 min per day, so 23 hours 45 min of this clock are the same as 24 hours of correct clock, i.e. 95/4 hours of this clock = 24 hours of correct clock. So, 114 hours of this clock = (24×4/95) × 114 hours of correct clock = 115.2 hours of correct clock, which is equal to 115 hours 12 minutes. So, the correct time must be 5:12 am. 16.(d) In a correct clock, the minute hand gains 55 min spaces over the hour hand in 60 min. To be together again, the minute hand must gain 60 min over the hour hand. 60 min are gained in (60/55) × 60 = 65(5/11) min But they are together after 63 min. So, gain in 63 min = 65 (5/11) – 63 = 2(5/11) = (27/11) min. So, gain in 24 hrs = [27/(11 × 63)] × 60 × 24 = 56(8/77) min.
  • 24. . 4 www.visionias.in ©Vision IAS 17.(c) Since it has been mentioned that Mohan was not born in February, so his date of birth can’t be 29th Feb. He will celebrate his next birthday on a Wednesday in the year by which the number of odd days becomes 5. By his birthday in 2017, there will be 1 odd day. By his birthday in 2018, there will be 2 odd days. By his birthday in 2019, there will be 3 odd days. By his birthday in 2020, there will be 5 odd days, as 2020 is a leap year. So, in 2020 he will celebrate his birthday on a Wednesday. 18.(c) According to the question, out of four, not more than two can be carnivores and at least two must be herbivores. It means that the possible number of carnivores and herbivores may be: (2, 2), (1, 3), (0, 4). Further it’s given that: Statement 1: Crocodile ↮ Giraffe and Rabbit Statement 2: Lion ↮ Tiger Statement 3: Cow ↮ Deer The combination given in option (c) is not possible because Crocodile cannot be included with Rabbit. 19.(d) Option (a) cannot be the answer because Cow cannot be included with Deer. Option (b) cannot be the answer because Lion cannot be included with Tiger. Option (c) cannot be the answer because Crocodile cannot be included with Giraffe. Hence, option (d) is the required answer. 20.(a) There are 3 scenarios: (1) If the top card is of the same suit as the base card, the Player pays twice the base amount to the Dealer. (2) If the top card is of the same colour as the base card (but not the same suit) then the Player pays the base amount to the Dealer. (3) If the top card happens to be of a different colour than the base card, the Dealer pays the base amount to the Player. Now Ghosh babu played four rounds of the game. Let us list down the total amount with him after each round. Round 1: 8 - 16 = -8 (from case 1) Round 2: (-8) + 10 + 10 =12 (from case 3) Round 3: (+12) + 6 - 6 =12 (from case 2) Round 4: (+12) + 8 —16 = 4 (from case 1) Hence, we can conclude that by round 2 and 3 he had the maximum profit of Rs 12. 21.(b) In the previous solution we have seen that maximum loss he incurred was Rs 8 at the end of 1st round. Hence, if he had Rs 8 he wouldn’t need to borrow money from anyone. 22.(d) With Rs. X initially, he is left with Rs. (X+4) at the end. Hence, X + 4 = 100 Or X = 96 23.(d) Statement 1 is incorrect. The passage says, primitive men imagined themselves to be living on a virtually illimitable plane, whereas presently the concepts of spherical earth and a closed sphere of human activity are being explored. We cannot conclusively say that this change in perception has been driven ONLY by rise in population; rather it may be because of the newly gained scientific knowledge regarding our planet.
  • 25. . 5 www.visionias.in ©Vision IAS Statement 2 is also incorrect. Author only says that primitive men imagined themselves to be living on a virtually illimitable plane. There is no direct indication towards whether or not they had the capability to adjust to changed circumstances. Secondly, the passage suggests that there has been a gradual change in perception of human beings. This would not be possible had primitive men lacked adjusting capability. 24.(d) Assumption (1) is correct. The passage mentions, “Freebies’ distributed to the public as a result of election promises are not new in India.” So, this assumption is correct. Assumption (2) is correct. The passage mentions, “The basic argument is that these are a waste of resources and place a burden on already stressed fiscal resources.” It implies a negative impact on the economy. The passage further mentions, “MGNREGA for instance has been another scheme which has been a lifeline for many during the pandemic and earlier.” It implies a positive impact on the society. So, this assumption is correct. Assumption (3) is correct. The passage mentions, “welfare schemes such as free or subsidized rations under the Public Distribution System (PDS), cooked meals under the mid-day meal scheme, supplementary nutrition through anganwadis, and work provided through the Mahatma Gandhi National Rural Employment Guarantee Act (MGNREGA). MGNREGA for instance has been another scheme which has been a lifeline for many during the pandemic and earlier. At a time when there are few employment opportunities, working under MGNREGA can guarantee some assured wages”. It implies, welfare schemes protect basic human rights to food and work, which fulfil the right to life with dignity and further contribute to human development. So, this assumption is correct. 25.(b) Assumption 1 is invalid. The given assumption is incorrect because the following line “Violations of the right to education may occur through the direct action of States parties (the act of commission) or through their failure to take steps required by law (the act of omission)”, illustrates that violations can happen through an act of omission as well, which means that even after integrating the right in national laws there is no surety that violations will not happen. So, to assume that integration will end violations is not correct. Assumption 2 is valid. As per the passage, “Violations of the right to education may occur through the direct action of States parties (the act of commission) or through their failure to take steps required by law (the act of omission)”, the state through an act of commission or omission can violate the right to education. So, this assumption is correct. 26.(d) We have to find the day on 1st April, 2002. 1st April, 2002 = (2000 years + time period from 1.1.2002 to 1.4.2002) 1600 years » 0 odd days 400 years » 0 odd days 2001 » 1 odd day In 2002, Jan + Feb + March = (31 + 28 + 31) = 90 days ≡ 6 odd days. So, Total number of odd days = (0 + 0 + 1 + 6) = 7 ≡ 0 On April 1, 2002 it was Monday. So, in the month of April 2002, Tuesday fell on 2nd, 9th, 16th, 23rd & 30th. 27.(c) The given figure is:
  • 26. . 6 www.visionias.in ©Vision IAS In rectangle EFHG, minimum 4 straight lines are there (i.e. EF, FH, GH, and EG) Similarly, in rectangle CDJI, there are minimum 4 straight lines. YT and MN are 2 straight lines. GO, GM, HP, HN are 4 straight lines. In triangle QKL, there are minimum 4 straight lines (i.e. QK, QR, QL and KL). In quadrilateral ACDB, minimum 5 straight lines are there (i.e. AC, CY, YD, DB and AB). Hence, minimum number of straight lines in the given figure = 4 + 4 + 2 + 4 + 4 + 5 = 23 Hence, option (c) is the correct answer. 28.(b) Let the number of pants purchased be x and the number of skirts purchased be y. It is given that, the number of skirts is two less than twice the number of pants purchased. So, y = 2x – 2 or 2x – y = 2 ……..(i) Also, the number of skirts is four less than four times the number of pants purchased So, y = 4x – 4 or 4x – y = 4 ……… (ii) Subtracting equation (i) from equation (ii), we get: x = 1 Putting this value of x in equation (i), we get: y = 0 Therefore, Sheetal bought 1 pant and 0 skirt. Hence, option (b) is the correct answer. 29.(b) R1 : 16 18 22 26 R2 : 8 9 10 13 R3 : 20 16 10 11 R4 : 28 25 22 ? The pattern is: R1 + R3 – R2 = R4 16 + 20 – 8 = 28 18 + 16 – 9 = 25 22 + 10 – 10 = 22 26 + 11 – 13 = 24 Hence, option (b) is the correct answer. 30.(d) Time is 2:20. Position of the hands: * Hour hand at 2 (nearly). * Minute hand at 4 Angle between 2 and 4 is 60 degrees. Angular distance travelled by the hour hand in 20 minutes is 10 degrees, since it turns through ½ degrees in a minute. Therefore, angle between the hands at 2:20 = 60 degrees - 10 degrees = 50 degrees 31.(d) At 11 O'clock, the hour hand is 5 spaces apart from the minute hand. During the next 60 minutes, i.e. between 11' O clock and 12' O clock the minute hand will move further away from the hour hand. It will eventually meet the hour hand exactly at 12 O'clock. 32.(b) After every 400 years, the calendar repeats. Thus, if 27th February, 2003 was a Thursday, then on 27th February, 1603 it must have been a Thursday too.
  • 27. . 7 www.visionias.in ©Vision IAS 33.(b) The clock loses 1% week-time during the first week. In a day there are 24 hours and in a week there are 7 days. Therefore, there are 7 × 24 = 168 hours in a week. If the clock loses 1% time during the first week, then it will show a time which is 1% of 168 hours less than 12 Noon at the end of the first week = 1.68 hours less. Subsequently, the clock gains 2% during the next week. The second week has 168 hours and the clock gains 2% time = 2% of 168 hours = 3.36 hours more than the actual time. As it lost 1.68 hours during the first week and then gained 3.36 hours during the next week, the net result will be a -1.68 + 3.36 = 1.68 hours net gain in time. So the clock will show a time which is 1.68 hours more than 12 noon two weeks from the time it was set right. 1.68 hours = 1 hour and 40.8 minutes, i.e. 1 : 40 P.M. 34.(d) Explanation for Questions 35 to 38: LR was the third movie to be screened on screen 4. PN was screened on two other screens prior to screen 3, and was screened on 1 immediately after it. was screened on 3. SCREEN 1 SCREEN 2 SCREEN 3 SCREEN 4 SCREEN 5 DAY 1 DAY 2 DAY 3 PN LR DAY 4 PN DAY 5 In all the screens, except screen 2, IR was screened just before IN. IN was released on screen 3 on day 2. SCREEN 1 SCREEN 2 SCREEN 3 SCREEN 4 SCREEN 5 DAY 1 IR DAY 2 IR IN DAY 3 IN PN LR IR DAY 4 PN IR IN DAY 5 IN Now, on screen 4 LR was screened two days after PN. SCREEN 1 SCREEN 2 SCREEN 3 SCREEN 4 SCREEN 5 DAY 1 IR PN DAY 2 IR IN ER DAY 3 IN PN LR IR DAY 4 PN IR IN DAY 5 IN On day 1, PN and ER were released on two consecutive screen numbers. SCREEN 1 SCREEN 2 SCREEN 3 SCREEN 4 SCREEN 5 DAY 1 IR PN ER DAY 2 IR IN ER DAY 3 IN PN LR IR DAY 4 PN IR IN DAY 5 IN Now, let’s fill up the gaps in screen 1. SCREEN 1 SCREEN 2 SCREEN 3 SCREEN 4 SCREEN 5 DAY 1 LR IR PN ER DAY 2 IR IN ER DAY 3 IN PN LR IR DAY 4 PN IR IN DAY 5 ER IN
  • 28. . 8 www.visionias.in ©Vision IAS Now, let’s fill up the gaps in screen 3 and day 4. SCREEN 1 SCREEN 2 SCREEN 3 SCREEN 4 SCREEN 5 DAY 1 LR IR PN ER DAY 2 IR IN ER DAY 3 IN PN LR IR DAY 4 PN LR ER IR IN DAY 5 ER LR IN Now, let’s fill up the gaps in screen 5 and day 5. SCREEN 1 SCREEN 2 SCREEN 3 SCREEN 4 SCREEN 5 DAY 1 LR IR PN ER DAY 2 IR IN ER LR DAY 3 IN PN LR IR DAY 4 PN LR ER IR IN DAY 5 ER IR LR IN PN Now, let’s fill up the gaps in screen 2. SCREEN 1 SCREEN 2 SCREEN 3 SCREEN 4 SCREEN 5 DAY 1 LR IN IR PN ER DAY 2 IR PN IN ER LR DAY 3 IN ER PN LR IR DAY 4 PN LR ER IR IN DAY 5 ER IR LR IN PN 35.(b) 36.(a) 37.(c) 38.(b) 39.(a) Statements 1 and 3 are correct. The lines “Drought will make water more expensive, which will likely affect the cost of raw materials and production. Climate volatility may force companies to deal with uncertainty in the price of resources for production, energy transport and insurance. And some products could become obsolete or lose their markets, such as equipment related to coal mining or skiing in an area that no longer has snow” reflect that options 1 and 3 are disadvantages of climate change. Statements 2 and 4 are incorrect. The lines “Whether in the U.S. or abroad, new regulations such as carbon pricing and subsidies that favour a competitor may affect a business’s bottom line. A company’s reputation could also suffer if it’s seen as doing something that hurts the environment”, this shows that carbon pricing is important for dealing with issues of climate change and also, if the companies are forced to think about the environment, then also, it is good for dealing with climate change. So, these are not disadvantages but benefits. 40.(a) Option (a) is correct. The lines “Drought will make water more expensive, which will likely affect the cost of raw materials and production. Climate volatility may force companies to deal with uncertainty in the price of resources for production, energy transport and insurance. And some products could become obsolete or lose their markets, such as equipment related to coal mining or skiing in an area that no longer has snow. Whether in the U.S. or abroad, new regulations such as carbon pricing and subsidies that favour a competitor may affect a business’s bottom line.”, illustrate the changes or impact of climate change on companies and the production of goods. Therefore, this option is the best crux of the passage. Option (b) is incorrect. The passage discusses the companies but to say that companies which do not follow climate governance will face a boycott of their products will not be correct. So, this option is beyond the scope of the passage. Option (c) is incorrect. This option in general seems to be correct but it is not based on the information given in the passage. The passage mainly focuses on the challenges of climate change from the
  • 29. . 9 www.visionias.in ©Vision IAS perspective of companies (not nations) and what impact will it have on their production methods. So, this is also not correct. Option (d) is incorrect. There is no discussion about the significance of sustainable development in coping with climate change. So, this option is beyond the scope of the passage. 41.(d) Option (a) is incorrect. The author is not pessimistic about sufferings in this world. Towards the conclusion, the author says people can be liberated from sufferings when they understand the impermanent nature of all their feelings, and stop craving for them. Thus, despite feelings being never- ending and ephemeral, sufferings can end. Option (b) is incorrect. In the very beginning the author says, the root of suffering is neither the feeling of pain nor of sadness nor even of meaninglessness. It is nowhere said that these feelings ONLY in combinations of two or three lead to sufferings. Rather, according to the author, real root of suffering is never-ending and pointless pursuit of ephemeral (short-lived) feelings. Option (c) is also incorrect. This statement on its own might be true. However, it cannot be inferred from this passage, as the author does not make any reference to Buddha. 42.(d) Angle between 3 and 8 on the clock = 240 - 90 = 150 degrees. The angle travelled by the hour hand from 3 = (40/60) x 30 = 20 degrees. Therefore, the net angle between the hour hand and the minute hand = 150 - 20 = 130 degrees. 43.(c) 44.(b) Explanation for Questions 45 to 48: Minimum possible number of times the operations could have been performed are as follows: 6:00 AM to 8:00 AM Operation I: Net addition of peaches = 113 – 103 = 10 Operation II: Number of peaches redistributed = [Increase in the number of peaches in boxes A, B and C – Net addition of peaches] = 18 – 10 = 8 Operation III: 0 Total = 18 8:00 AM to 10:00 AM Operation I: Net addition of peaches = 125 – 113 = 12 Operation II: Number of peaches redistributed = [Increase in the number of peaches in boxes A, B and C – Net addition of peaches] = 17 – 12 = 5 Operation III: 0 Total = 17 10:00 AM to 12:00 PM Operation I: Net addition of peaches = 0 Operation II: Number of peaches redistributed = 1 (say from D to B) Operation III: Peaches thrown away = (29 - 26) + (40 - 39) = 4 Total = 5 12:00 PM to 2:00 PM Operation I: Net addition of peaches = 134 – 121 = 13 Operation II: Number of peaches redistributed = [Increase in the number of peaches in boxes A and D – Net addition of peaches] = 23 - 13 = 10 Operation III: 0 Total = 23 So, total number of operations performed by Kumar = 18 + 17 + 5 + 23 = 63. 45.(c) 46.(b)
  • 30. . 10 www.visionias.in ©Vision IAS 47.(c) From 6:00 AM to 8:00 AM: 8 – (25 – 23) – (31 – 27) = 2 From 8:00 AM to 10:00 AM : 5 – (29 – 25) = 1 From 10:00 AM to 12:00 PM: 0 From 12:00 PM to 2:00 PM: 0 Therefore, the required answer = 2 + 1 = 3. 48.(a) Required number = 8 + 5 + 1 + 10 = 24. 49.(b) Assumption 1 is invalid. The given statement is extreme. As per the lines “This approach includes educating people about dietary changes and helping them achieve optimum levels of nutrition. An important part of this is educating children at schools and people in shared spaces such as workplaces. The main aim of this approach is preventive”, community nutrition is preventive but to assume that it is not possible to overcome malnutrition by obesity would not be correct because there is no such relation being discussed in the passage. Assumption 2 is invalid. The lines “We currently have no significant programmes that target malnutrition affected by obesity”, merely show that there are no significant programmes to tackle malnutrition by obesity, but to assume that the absence of programmes is the primary cause of malnutrition would not be correct. Also, the primary cause of malnutrition is not discussed in the passage. So, this assumption is not correct. Assumption 3 is valid. The given statement is about replacing packaged and high-salt food with water- rich live foods because the lines “With the rise of packaged food, food with high salt and sugar content in food obesity-related malnutrition is on the rise. India is now facing a double whammy of malnutrition, with the younger population being affected by undernutrition and the older population affected by malnutrition related to obesity”, show that packaged or high-salt foods are the cause of malnutrition. Therefore, consuming fresh and nutrition rich food will help fight malnutrition. 50.(b) Option (a) is incorrect. The given option states that community nutrition is the best approach. However, such a conclusion can only be made when two approaches are compared which is not done in the passage. So, to conclude that the community nutrition approach is the best would not be correct. Option (b) is correct. The passage is mainly based on the theme of the use of a community nutrition approach to fight malnutrition by obesity. The lines “We currently have no significant programmes that target malnutrition affected by obesity. As a country, we could take the community nutrition approach where nutrition is improved at the community level”, show that India needs to focus on handling malnutrition by obesity and the community nutrition approach is a promising way to do it. Option (c) is incorrect. The option talks about a change of government approach in fighting malnutrition by obesity. However, the main theme of the passage is not about issues with government programmes but the use of a community nutrition approach in fighting malnutrition by obesity. So, this option is not the best message of the author. Option (d) is incorrect. The given option is beyond the scope of the passage because there is no mention of companies producing packaged foods, and also what needs to be done on their behalf. There is no discussion about the regulation of such companies. So, this is not the best message of the author. 51.(b) Inference 1 is incorrect. The given statement is partially correct. The assertion about being dangerous to the environment is correct. However, to say that it has some potential benefits is not correct because it is not based on any information in the passage. So, on whole, the inference is not correct. Inference 2 is correct. The lines “Invasive mammalian predators are therefore important drivers of irreversible loss of phylogenetic diversity worldwide. That most impacted species are insular indicates that management of invasive predators on islands should be a global conservation priority”, illustrating that the island being an insular habitat contains species which are most impacted by invasive predators. Therefore, island species are most vulnerable and need conservation on a priority basis.
  • 31. . 11 www.visionias.in ©Vision IAS 52.(d) Option (a) is incorrect. The given option is beyond the scope of the passage because there is no relation drawn between the survival of the human race and the conservation of biodiversity which is being discussed in the passage. Option (b) is incorrect. The given option is not correct because there is no discussion about the funding of islands for conserving biodiversity there or fighting invasive species. So, this option is beyond the scope of the passage. Option (c) is incorrect. This given statement is not based on the information in the passage. Also, it is extreme because to conclude that invasive species can grow anywhere would not be correct without any supporting information. Therefore, this option is beyond the scope of the passage. Option (d) is correct. The lines “Invasive predators are implicated in 87 bird, 45 mammals, and 10 reptile species extinctions - 58% of these groups’ contemporary extinctions worldwide. These figures are likely underestimated because 23 critically endangered species that were assessed are classed as “possibly extinct”, show that invasive predators are the cause of extinction for many species. Extinction of any species from the ecosystem alters its species diversity. Therefore, the given assertion in the statement is correct and it is the best crux of the passage. 53.(d) Let the number of keepers be x. Then, total number of feet = 2 × 50 + 4 × 45 + 4 × 8 + 2x = 2x + 312. Now, (2x + 312) = (103 + x) + 224 or x = 15. 54.(d) The seven pieces consist of 6 smaller equal pieces and one half cake piece. Weight of each small piece = 20 g. So, total weight of the cake = [2 × (20 × 6)]g = 240 g. 55.(d) Total relative angular distance to be covered by minute hand = 7 ×30° = 210° Minute hand gains 11/2° over hour hand in 1 minute Time required to make 210° gain = (2/11)×210 = 420/11 = 38(2/11) min. past 1 56.(a) 57.(a) See the third row. First box has 2 circles & 0 triangles, 2nd has 1 circle (decrease by 1) & 1 triangle (increase by 1). So, 3rd box must have 0 circles ad 2 triangles. Similar pattern is followed in all rows. You can also solve this by considering columns instead of rows. 58.(b) It is given that 28th August, 1946 was a Wednesday. From 28th August, 1946 to 28th August 1961, we have 4 leap years and 11 normal years. So, the number of odd days = 11×1 + 4×2 = 19 = 5 Now we have to move from 28th August to 31st August, i.e. 3 more odd days. So, total number of odd days = 5 + 3 = 8 = 1 So, 31st August 1961 would be Wednesday + 1 = Thursday 59.(b) Palmyra < Date palm; Date palm < Queen palm; Queen palm > Caryota After combining first two statements, we get: Palmyra < Date palm < Queen palm and Caryota < Queen palm [Position of Caryota is not certain but it is lesser than Queen palm for sure, so it can be at any place below Queen palm] Of the given options, only option (B), i.e. Palmyra < Queen palm, is definitely always true.
  • 32. . 12 www.visionias.in ©Vision IAS 60.(c) From condition 4, we can get the exact position of class IV (to which Hindi teacher went) and the exact position of the class of Science teacher. Condition 5 says that there are four classes to the left of class l. So, one class will be to the right of class I. It is also given that class IV and lll are not adjacent to each other. Hence we get: As per condition 3, English teacher went to the class to the immediate left of class I. As per condition 2, Math teacher went to class III which is adjacent to class II. Thus, class III cannot lie to the right of class I. Hence, class III will lie second to the left of class I. Now, as per condition 1, Drawing teacher went to the class to the immediate left of class V and to the immediate right of class ll. Therefore, class V will lie on rightmost side and Drawing teacher will take class I. Hence, final arrangement will be: We can see that, Math teacher went to class III. 61.(a) Science teacher went to class VI. 62.(c) Class I is to the immediate right of class II, and is taken by Drawing teacher. 63.(b) Assumption 1 is invalid. The given assumption is not correct because the lines “Currently, there are U.S. public programs that promote renewable energy options and recommendations in conserving fuel and energy for farms, efficient use of fertilizers and pesticides, soil conservation measures, and energy- efficient methods of growing and transporting foods”, talk about replacing fossil fuels with renewable energy. It means that agriculture in the U.S is not completely dependent on fossil fuels. Therefore, to assume that its future is bleak would not be correct. Assumption 2 is valid. The lines “Currently, there are U.S. public programs that promote renewable energy options and recommendations in conserving fuel and energy for farms, efficient use of fertilizers and pesticides, soil conservation measures, and energy-efficient methods of growing and transporting foods, but those programs will need high levels of participation to curb both climate impacts as well as address air and water quality issues associated with traditional agricultural practices”, show that along with replacing fossil fuels with renewable energy, it is important to ensure high levels of participation to curb climate change and address air and water quality issues of agriculture. So, it is correct to assume that only fossil-fuel replacement may not address the air and water pollution caused due to agriculture. 64.(a) All Statements are correct. The line “Direct energy use in agriculture is primarily from fossil fuels used to operate automobiles and machinery for preparing fields, planting, and harvesting crops, applying chemicals, and transporting inputs and outputs to and from the market”, illustrates that all the given options are the possible uses of energy in agriculture. 65.(d) Statement 1 is correct. It is mentioned in the passage that the JKEAC had looked at the mining blocks in the most recent October 2021 Google Image. It was discovered that extensive unlawful mining has
  • 33. . 13 www.visionias.in ©Vision IAS significantly reduced the availability of minerals. Therefore, it can be concluded that the government is getting help from technology for environmental protection. Hence, it is a correct statement. Statement 2 is correct. It is mentioned in the passage that in Kashmir and Himachal Pradesh, Kuhls are conventional irrigation systems that are run by the local community. Further, it says there are also irrigation Kuhls in the mining area and mining is detrimental to the flow of water. Hence, it is a correct statement. Statement 3 is correct. The passage says that the Hokersar wetland is fed by the Shaliganga Nallah, and mining activity may harm the water's natural flow and quality as it enters Hokersar. Hence, it is a correct statement. 66.(d) Statement 1 is correct as the passage mentions, “The rapidly snowballing loss in diversity is largely triggered and fuelled by new age emergencies and interventions, like climate change, change in atmospheric composition and the spread of invasive alien species.” Hence statement 1 is correct. Statement 2 is correct as the passage mentions, “The rapidly snowballing loss in diversity is largely triggered and fuelled by new age emergencies and interventions, like climate change, change in atmospheric composition and the spread of invasive alien species.” and also warming of temperature above pre-industrial levels that has impacted agriculture and contributed to the migration of species. Hence statement 2 is correct. Statement 3 is correct as the passage mentions, “In just the last three centuries, global forest areas have shrunk by 40 per cent. Every year, to meet the timber needs from natural sources, the Earth is stripped of 100 million trees.”, which clearly indicates deforestation. Its impact has been the risk of faster extinction in the plant kingdom. Hence, statement 3 is correct too. 67.(c) Originally, let number of women = x. Then number of men = 2x. So, in city Y: (2x – 10) = (x + 5) or x = 15. So, total number of passengers in the beginning = (x + 2x) = 3x = 45. 68.(d) Given equation: 5# × 3#0 ÷ #0 – 3#3 + #3 = 3# Let us use the options: From option (a), # = 4 54 × 340 ÷ 40 – 343 + 43 ≠ 34 (not satisfied) From option (b), # = 5 55 × 350 ÷ 50 – 353 + 53 ≠ 35 (not satisfied) From option (c), # = 8 58 × 380 ÷ 80 – 383 + 83 ≠ 38 (not satisfied) From option (d), # = 6 56 × 360 ÷ 60 – 363 + 63 = 36 (satisfied) Hence, option (d) is the correct answer. 69.(b) x weeks x days = (7x + x) days = 8x days. 70.(c) If yesterday were tomorrow, then today would be Friday. So, day before yesterday was Friday. So, today is a Sunday. Explanation for Questions 71 to 74: Consider members of groups C and E. From I and II, R and T are not Y’s family members. X and Y are members of group E. So, X is not from Y’s family. So, group C has member V from Y’s family. Q and Y & Z are members of group B. O and V & Z are members of group D. So, O and Q are not members of Y or Z’s family. Also, R and Y are not from the same family. So, S must be from the family
  • 34. . 14 www.visionias.in ©Vision IAS of Y and V, and R must be from the family of Z (inferred from data in group A). Also, from III and IV, Y, V and S belong to either Mishra's or Malhotra's. Group A O Q R (family 2) S (family 1) Group B Q W Y (family 1) Z (family 2) Group C R (family 2) T V (family 1) X Group D O U V (family 1) Z (family 2) Group E P U X Y (family 1) Now consider groups A and D. Neither Q nor Z are from same family as O. So, among members of group B, Q, Z and Y are not from Ahuja family. So, W has to be from Ahuja family. R and V are not Ahuja family members. Also, T is not from the Ahuja’s. So, X must be from the Ahuja’s (inferred from group C). Thus, Ahuja family: O, W and X. Group A O (Ahuja) Q R (family 2) S (family 1) Group B Q W (Ahuja) Y (family 1) Z (family 2) Group C R (family 2) T V (family 1) X (Ahuja) Group D O (Ahuja) U V (family 1) Z (family 2) Group E P U X (Ahuja) Y (family 1) Consider remaining members of groups A, B, C, and D. It can be concluded that R, Z and P are members of the same family, while Q, T and U are from one family too. Group A O (Ahuja) Q (family 3) R (family 2) S (family 1) Group B Q (family 3) W (Ahuja) Y (family 1) Z (family 2) Group C R (family 2) T (family 3) V (family 1) X (Ahuja) Group D O (Ahuja) U (family 3) V (family 1) Z (family 2) Group E P (family 2) U (family 3) X (Ahuja) Y (family 1) Now, from IV, Sharma family is family 2: R, Z and P. Q, T and U belong to family 3, i.e., either Malhotra's or Mishra's. Group A O (Ahuja) Q (family 3) R (Sharma) S (family 1) Group B Q (family 3) W (Ahuja) Y (family 1) Z (Sharma) Group C R (Sharma) T (family 3) V (family 1) X (Ahuja) Group D O (Ahuja) U (family 3) V (family 1) Z (Sharma) Group E P (Sharma) U (family 3) X (Ahuja) Y (family 1) 71.(b) X is from the same family as W. 72.(c) Both Z and P are from the Sharma family. 73.(d) 74.(d) We can determine that O, W and X are from Ahuja family, and P, R and Z are from Sharma family. 75.(c) When Rahul was born: His brother’s age = 6 years, His father’s age = (6 + 32) = 38 years. His mother’s age = (38 – 3) years = 35 years. His sister’s age = (35 – 25) years = 10 years. 76.(d) Statement 1 is incorrect as the passage mentions “Especially in countries that score high on the World Happiness Index (WHI), there is a strong link between the feeling of needing to be happy and the extent to which people actually experience feelings such as sadness, gloom, fatigue or anxiety." It does not mean
  • 35. . 15 www.visionias.in ©Vision IAS that the link between needing to be happy and negative emotions is weak in countries that score low on WHI. There is insufficient information to arrive at the conclusion. Hence statement 1 is incorrect. Statement 2 is incorrect as the passage mentions that social media, in self-help books and advertisements create an image of "happiness norm". Society has created an image of what it means to be happy. However, it does not imply that it actually leads to happiness in life. Hence statement 2 is incorrect. 77.(c) Option (a) is incorrect as the passage mentions “autonomy goes beyond individualism and self- expression and involves the exercise of duties, obligations, and respect for other moral agents and their own rights." The passage does not mention whether the patient has the right to treatment. Hence statement 1 is incorrect. Option (b) is incorrect as although the passage emphasises the autonomy of the patient, whether such right has to remain inviolable at all the times is not clear from the passage. Further, the main message of the passage is balance between patient's rights and doctor's rights. Hence statement 2 is incorrect. Option (c) is correct as the passage mentions "Ideas of autonomy have not only been related to the individual self-expression of the patient; indeed, they have been used to validate arguments for clinical independence and discretion. O’Neill has argued, following Kant, that autonomy goes beyond individualism and self-expression and involves the exercise of duties, obligations, and respect for other moral agents and their own rights." Therefore, the emphasis is on balance between patient's rights and doctor's rights. Hence statement 3 is correct. Option (d) is incorrect as although the statement is factually true in this context, it does not form the central argument of the passage, as stated above. Hence statement 4 is incorrect. 78.(a) Statement 1 is correct. The passage says that the new visa system was put into place in an effort to "enhance the quality of life and make the experience of living, working, and investing in the UAE a happy and cheerful one." Therefore, it can be inferred that the new visa system seeks to increase investment in UAE. Statement 2 is incorrect. As mentioned in the passage, with the introduction of a flexible, multiple-entry tourist visa with a five-year validity period, they can stay in the UAE for up to 90 days nonstop. Hence, it is not a correct statement. 79.(b) Statement 1 is incorrect. According to the passage, a census or survey is the best approach to determine whether the majority of Indian families are still considered "joint families”, and if one wants to compare the situation of women in nuclear and combined families, participant observation, case studies, and interviews may all be useful techniques. Hence it is not a correct statement. Statement 2 is correct. As mentioned in the passage there are numerous methodologies in sociology given the multiplicity of perspectives and truths. The path to social truth is not one and the same. Hence, it is a correct statement. 80.(b) Option (a) is incorrect. According to the passage, it is a partially incorrect statement. It is only a part of the central argument which says that multiple truths exist in sociology, simultaneously. Hence, it is not a correct statement. Option (b) is correct. It is the most important implication of the passage. The passage says that there are numerous methodologies in sociology given the multiplicity of perspectives and truths. The path to social truth is not one and the same. The passage revolves around this central argument. Hence, it is the correct statement. Option (c) is incorrect. It is not mentioned anywhere in the passage. Hence, it is not a correct statement. Option (d) is incorrect. In the sociological study, quantitative and qualitative methods are used. But among the two which one is more useful and used more is not mentioned. Hence, it is not a correct statement. Copyright © by Vision IAS All rights are reserved. No part of this document may be reproduced, stored in a retrieval system or transmitted in any form or by any means, electronic, mechanical, photocopying, recording or otherwise, without prior permission of Vision IAS.